Forum: Offtopic Vergleichen von Grössen


von (prx) A. K. (prx)


Lesenswert?

Frank M. schrieb:
> So geht man wissenschaftlich vor.

Wissenschaftliche Vorgehensweise führt zu Voreingenommenheit.

von Johann L. (gjlayde) Benutzerseite


Lesenswert?

Herrscht denn wenigstens Übereinstimmung darüber, daß das MM-Experiment 
mit folgenden Theorien vereinbar ist?

1) Emissionstheorie
2) Vollständigge Äthermitführung
3) Bindls Theorie
4) Spezielle Relativitätstheorie

Das MM-Experiment ist also nicht in der Lage, zwischen 1), 2), 3) und 4) 
zu unterscheiden.  Warum wird dann an diesem Experiment geklebt wie 
Kaugummi?

von Frank M. (ukw) (Moderator) Benutzerseite


Lesenswert?

Kurt Bindl schrieb:
> Heisst das das du nicht in der Lage bist das was ich schreibe
> nachzuvollziehen

Ja.

> (weil du wohl nur Formeln kennst),

Nein.

> dann ist es wirklich
> schade um die Uhren die du an die Wand gehängt hast, denn wem dieses
> selbstverständliche Gedankengut fehlt der wird auch nicht kapieren wie
> das mit den beiden Signalen beim MMI einhergeht.

Genau. Deshalb sprichst Du hier nur mit Doofsäcken wie mir, welche Dein 
Geschwätz einfach nicht verarbeiten können.

Der Grund ist einfach:

Du gehst davon aus, dass alle "Deine Sprache" sprechen. Dem ist nicht 
so. Deine Sprache, die total von der Sprache abweicht, welche 
Naturwissenschaftler sprechen, versteht nur Dein geniales Gehirn - sonst 
keiner.

Du musst Dich an die Sprache der Naturwissenschaften anpassen, nicht 
umgekehrt, wenn Du mitreden willst. Sonst mach das mit Dir selbst im 
stillen Kämmerlein aus.

Also: Was für ein Produkt von was meinst Du?

von Frank M. (ukw) (Moderator) Benutzerseite


Lesenswert?

A. K. schrieb:
> Wissenschaftliche Vorgehensweise führt zu Voreingenommenheit.

Wenn Du jetzt neben mir gestanden wärst, hättest Du Dich auf einen 
(freundschaftlich gemeinten) Rempler gefasst machen können ;-)

von Kurt B. (kurt-b)


Lesenswert?

Johann L. schrieb:
> Herrscht denn wenigstens Übereinstimmung darüber, daß das MM-Experiment
> mit folgenden Theorien vereinbar ist?
>
> 1) Emissionstheorie
> 2) Vollständigge Äthermitführung
> 3) Bindls Theorie
> 4) Spezielle Relativitätstheorie
>
> Das MM-Experiment ist also nicht in der Lage, zwischen 1), 2), 3) und 4)
> zu unterscheiden.  Warum wird dann an diesem Experiment geklebt wie
> Kaugummi?

Weil man in es alles hineininterpretrieren kann was gerade -sinnvoll- 
(nützlich) erscheint.

 Kurt

Ich kann es nehmen um zu -beweisen- dass die Erde den LLBezug bildet.
Derdort kann es verwenden um seinen Äther mitgezogen zu sehen.
Drüben einer der die Lichtinvarianz bestätigt sieht.
Jener sieht -Emission-.
Sonsteiner das der Schnee dunkelblaurot ist.
...

Solange es nicht sicher ist dass es auch so funzt wie angenommen (weil 
ich das nicht gegeben sehe (darum das mit der Phasenanpassung)) können 
auch keine brauchbaren Schlüsse daraus gezogen werden.

 Kurt

von Frank M. (ukw) (Moderator) Benutzerseite


Lesenswert?

Kurt Bindl schrieb:
> Solange es nicht sicher ist dass es auch so funzt wie angenommen (weil
> ich das nicht gegeben sehe (darum das mit der Phasenanpassung)) können
> auch keine brauchbaren Schlüsse daraus gezogen werden.

Schwall ins All? Was schreibst Du da? Was ist nicht sicher, dass was 
nicht funktioniert ("funzt" gibt es nicht)? Was nimmst Du an? Was siehst 
Du als nicht gegeben? Warum die Phasenanpassung? Welche Schlüsse sind 
nicht brauchbar? Wer soll die Schlüsse ziehen?

Merkst Du eigentlich, dass Du hier öffentlich lediglich Selbstgespräche 
brabbelst, die keiner verstehen kann?

: Bearbeitet durch Moderator
von Kurt B. (kurt-b)


Lesenswert?

Frank M. schrieb:

>
> Du musst Dich an die Sprache der Naturwissenschaften anpassen, nicht
> umgekehrt, wenn Du mitreden willst. Sonst mach das mit Dir selbst im
> stillen Kämmerlein aus.
>
> Also: Was für ein Produkt von was meinst Du?

Produkt ist ja vorsorglich in -Produkt- gesetzt.

Das "Produkt" das aus dem Ausgang rauskommt ist erstellt aus zwei 
frequenzgleichen, aber phasenverschobenen Signalen, zwei Signalen von je 
100MHz.

Diese beiden Signale koppeln ganz leicht auf den Ausgangsschwingkreis 
rüber, das bedeutet dass dieser Schwingkreis (den unsichtbaren 
Kondensator werd ich noch dazu malen) beide Signale, stark abgeschwächt 
bekommt.
Er müsste sich also entscheiden welches er nimmt, das geht aber nicht 
denn beide schreien: ich bin da.
Es kommt zu einer Schwingung deren Frequenz 100 MHz beträgt, jedoch 
keiner gleicht, sondern dazischen_drin ist.
Einem Signal von 100 MHz mit einer Phasenlage von 45° zur 
Grundschwingung.

Selbst wenn man die beiden Eingangssignale vertauscht kommt immer das 
selbe raus, immer 45°.

Nun sehen wir uns das MMI an, es bietet ebenfalls die Voraussetzungen 
dass so ein "Produkt" entsteht, nämlich die schwache Kopplung.

Selbst wenn die beiden Arme getauscht werten (90 Grad Drehung) das 
Signal das mit sich selbst interferiert ist immer das gleiche.

Darum -trau- ich einem MMI, egal ob alt oder neu, nicht über den Weg.


 Kurt

von Frank M. (ukw) (Moderator) Benutzerseite


Lesenswert?

Kurt Bindl schrieb:
> Diese beiden Signale koppeln ganz leicht auf den Ausgangsschwingkreis
> rüber, das bedeutet dass dieser Schwingkreis (den unsichtbaren
> Kondensator werd ich noch dazu malen) beide Signale, stark abgeschwächt
> bekommt.

Soweit klar. Wie Kurt auf der Schaukel.

> Er müsste sich also entscheiden welches er nimmt, das geht aber nicht
> denn beide schreien: ich bin da.

Häh? Welches er nimmt??? Das kann Kurt in der Mitte der Schaukel, welche 
von zwei Seiten phasenversetzt angeregt wird, gar nicht entscheiden!

Ich verstehe Dein Brabbeln nicht. Erkläre das bitte, aber tue dabei 
nicht so, als ob Deine linke Gehirnwindung es Deiner rechten erzählt. 
Die Methode taugt nichts.

> Es kommt zu einer Schwingung deren Frequenz 100 MHz beträgt, jedoch
> keiner gleicht, sondern dazischen_drin ist.

Achso? Quellennachweis?

> Einem Signal von 100 MHz mit einer Phasenlage von 45° zur
> Grundschwingung.

Wieso 45° und nicht 42°? Wo steht das? Wie kann ich das nachrechnen? Wie 
ist das verifizierbar?

Sorry, Du kannst nicht erwarten, dass ein anderer Deine Kurzschlüsse, 
die ausschließlich in Deinem Gehirn sitzen, nachvollziehen kann. Da 
musst Du dem anderen schon etwas anderes an die Hand geben.

> Selbst wenn man die beiden Eingangssignale vertauscht kommt immer das
> selbe raus, immer 45°.

Warum? Könnte es auch sein, dass es beides mal 42° sind?

> Nun sehen wir uns das MMI an, es bietet ebenfalls die Voraussetzungen
> dass so ein "Produkt" entsteht, nämlich die schwache Kopplung.

Bei einem "Produkt" multipiliziert der Physiker eine Größe mit einer 
anderen. Bitte wähle einen anderen Begriff, wenn Du NICHT das 
mathematische Produkt meinst.

Eine schwache Kopplung? Beim MM-Experiment? Wo koppelt da was schwach 
miteinander?

> Selbst wenn die beiden Arme getauscht werten (90 Grad Drehung) das
> Signal das mit sich selbst interferiert ist immer das gleiche.

Welche Arme? Die von Kurt auf der Schaukel?

> Darum -trau- ich einem MMI, egal ob alt oder neu, nicht über den Weg.

Das ist keine Frage von trauen oder gar Vertrauen.

: Bearbeitet durch Moderator
von Kurt B. (kurt-b)


Lesenswert?

Frank M. schrieb:

>
> Ich verstehe Dein Brabbeln nicht. Erkläre das bitte, aber tue dabei
> nicht so, als ob Deine linke Gehirnwindung es Deiner rechten erzählt.
> Die Methode taugt nichts.
>
>> Es kommt zu einer Schwingung deren Frequenz 100 MHz beträgt, jedoch
>> keiner gleicht, sondern dazischen_drin ist.
>
> Achso? Quellennachweis?
>
>> Einem Signal von 100 MHz mit einer Phasenlage von 45° zur
>> Grundschwingung.
>
> Wieso 45° und nicht 42°? Wo steht das? Wie kann ich das nachrechnen? Wie
> ist das verifizierbar?

Entschuldige, aber ich erwarte von Leuten die in einem -Technikerforum- 
schreiben wo es auch um HF geht, zumindest ein Mindestverständniss für 
Vorgänge im HF-Bereich.

Wenn du das mit der schwachen Kopplung der beiden Schwingkreise zum 
dritten nicht verstehst dann soll halt mal einer der OMs hier dir das 
versuchen zu vermitteln, vielleicht spricht derjenige deine Sprache wenn 
du schon die in diesem Zweig übliche nicht verstehst.


Formeln (willst du sehen)? wozu?

(X1 + X2) / 2 ergibt doch höchstwahrscheinlich 45, oder nicht?
Da brauchts doch keines Rätselns.

(X1 steht für die Phasenlage des 100MHz Signals am Eingang1 (E1),
X2 für die Phasenlage des 100MHz Signals am Eingang2 (E2))

X1 = 0
X2 = 90


Vielleicht hilft es dir wenn ich dir sage das diese, -von mir versucht 
zu beschreiben- Umstände dazu führen dass das Ergebnis beim MMI immer 
Null ist, egal was wirklich anliegt, bzw. wohin man es auch dreht.

 Kurt

von Frank M. (ukw) (Moderator) Benutzerseite


Lesenswert?

Kurt Bindl schrieb:
> (X1 + X2) / 2 ergibt doch höchstwahrscheinlich 45, oder nicht?
> Da brauchts doch keines Rätselns.

Wie kommst Du denn darauf, dass die Phasenverschiebung konstant ist? Síe 
könnte genausogut zwischen 0° und 90° wandern. Denk nur mal an 
Schwebungen bei gekoppelten Pendeln.

> (X1 steht für die Phasenlage des 100MHz Signals am Eingang1 (E1),
> X2 für die Phasenlage des 100MHz Signals am Eingang2 (E2))
>
> X1 = 0
> X2 = 90

Ja und? Redest Du hier jetzt hier von einem arithmetischen Mittel? 
Machst Du es Dir angesichts eines dynamischen Vorgangs nicht ziemlich 
einfach, von konstanten 45° zu sprechen? Die Konstanz kann ich leider 
nicht nachvollziehen. Die von mir angesprochenen 42° sind zu bestimmten 
Zeitpunkten genauso möglich wie Deine 45°.

> Vielleicht hilft es dir wenn ich dir sage das diese, -von mir versucht
> zu beschreiben- Umstände dazu führen dass das Ergebnis beim MMI immer
> Null ist, egal was wirklich anliegt, bzw. wohin man es auch dreht.

Was glaubst Du, was heraus kommt, wenn Du MMI auf dem Mond wiederholst? 
Dass da Dein sog. "Träger" dann dieses Mal an den Mond "getackert" 
(Deine Worte!) ist? Wenn ja, wieso bewegt sich dann der Mond noch 
relativ zur Erde?

Komm mal auf den Punkt. Deine Ansichten (die lediglich 
Gedankenexperimenten entspringen) kann man genauso durch reine Gedanken 
in der Luft zerfetzen. So kommst Du nicht weiter. Du musst schon was 
handfestes vorweisen.

: Bearbeitet durch Moderator
von (prx) A. K. (prx)


Lesenswert?

Kurt Bindl schrieb:
> Vielleicht hilft es dir wenn ich dir sage das diese, -von mir versucht
> zu beschreiben- Umstände dazu führen dass das Ergebnis beim MMI immer
> Null ist, egal was wirklich anliegt, bzw. wohin man es auch dreht.

Weisslichtinterferometrie ist also prinzipiell unmöglich?

von Guido B. (guido-b)


Lesenswert?

Interessante Methode, um die komplexe Rechnung zu Umgehen! 8-)

von Frank M. (ukw) (Moderator) Benutzerseite


Lesenswert?

A. K. schrieb:

> Weisslichtinterferometrie ist also prinzipiell unmöglich?

Breitbandiges Weisslicht besteht ja bekanntlich aus ganz vielen Farben. 
Und ganz bestimmten Farben traut Kurt einfach nicht über den Weg.

von Kurt B. (kurt-b)


Lesenswert?

A. K. schrieb:
> Kurt Bindl schrieb:
>> Vielleicht hilft es dir wenn ich dir sage das diese, -von mir versucht
>> zu beschreiben- Umstände dazu führen dass das Ergebnis beim MMI immer
>> Null ist, egal was wirklich anliegt, bzw. wohin man es auch dreht.
>
> Weisslichtinterferometrie ist also prinzipiell unmöglich?

Weisslichtinterferometrie zeigt, so wie jede Linse es auch macht, dass 
Frequenz- und winkelabhängige Brechung stattfindet, dass also das Licht 
das durch Glas, und jede andere Materie, durchgeht, beeinflusst wird, 
sich gegenseitig beeinflusst.

Und das Sagnacinterferometer muss sogar eine Grunddrehung oder 
Schwenkung erfahren damit der Einphaseffekt nicht zur Geltung kommt bzw. 
rausgerechnet werden kann.

Es liegt eine Beeinflussung vor, das dürfte doch wohl klar sein.
Diese Beeinflussungen sind aber in keinem MMI-Ergebnis berücksichtigt.

 Kurt

von Kurt B. (kurt-b)


Lesenswert?

Frank M. schrieb:

>
> Ja und? Redest Du hier jetzt hier von einem arithmetischen Mittel?
> Machst Du es Dir angesichts eines dynamischen Vorgangs nicht ziemlich
> einfach, von konstanten 45° zu sprechen? Die Konstanz kann ich leider
> nicht nachvollziehen. Die von mir angesprochenen 42° sind zu bestimmten
> Zeitpunkten genauso möglich wie Deine 45°.
>

Willst du nicht oder kannst du nicht verstehen?

Wenn es bei dir unbedingt 42° sein müssen dann verändere halt die 
Kopplung der beiden Eingangskreise zum Ausgangskreis, dann kannst du, 
anhand des Koppelfaktors, jeden Winkel (0..45°) des Ausgangssignals zum 
Quellsignal einstellen.

Und rechnen kannst du dann auch so viel du willst.
Ob du dann das was ich schreibe und übers MMI stülpe dann besser 
verstehst weiss ich nicht.


 Kurt

von Frank M. (ukw) (Moderator) Benutzerseite


Lesenswert?

Kurt Bindl schrieb:
> Es liegt eine Beeinflussung vor, das dürfte doch wohl klar sein.

Deine Rhetorik in Ehren, aber da ist nix klar. In welchem quantivtativen 
Rahmen bewegen sich diese "Beeinflussungen". Kannst Du eine 
Fehlerabschätzung vornehmen?

von Frank M. (ukw) (Moderator) Benutzerseite


Lesenswert?

Kurt Bindl schrieb:
> Wenn es bei dir unbedingt 42° sein müssen dann verändere halt die
> Kopplung der beiden Eingangskreise zum Ausgangskreis, dann kannst du,
> anhand des Koppelfaktors, jeden Winkel (0..45°) des Ausgangssignals zum
> Quellsignal einstellen.

Was hat Dein Winkel von 45° mit den beiden Uhren an der Wand zu tun, die 
sich einphasen werden?

> Und rechnen kannst du dann auch so viel du willst.

Du gibst mir ja nichts zum Rechnen (sprich Verifizieren) an die Hand. 
Also lügst Du.

> Ob du dann das was ich schreibe und übers MMI stülpe dann besser
> verstehst weiss ich nicht.

Ich verstehe noch nichtmals Deine Zeichensetzung und muss daher Deine 
Sätze (wie diesen) immer zweimal lesen, bevor ich überhaupt einen Sinn 
verstehe. Du machst es einem wirklich nicht einfach.

Ich halte Dich immer mehr für einen handfesten Lügner. Aber damit bin 
ich ja nicht der erste.

Und Du hast aus dem von Dir zitierten Posting eine Frage ignoriert, 
daher nochmal:

Was glaubst Du, was heraus kommt, wenn Du MMI auf dem Mond wiederholst?
Dass da Dein sog. "Träger" dann dieses Mal an den Mond "getackert"
(Deine Worte!) ist? Wenn ja, wieso bewegt sich dann der Mond noch
relativ zur Erde?

: Bearbeitet durch Moderator
von Johann L. (gjlayde) Benutzerseite


Lesenswert?

Kurt Bindl schrieb:
> Selbst wenn man die beiden Eingangssignale vertauscht kommt immer das
> selbe raus, immer 45°.
>
> Nun sehen wir uns das MMI an, es bietet ebenfalls die Voraussetzungen
> dass so ein "Produkt" entsteht, nämlich die schwache Kopplung.
>
> Selbst wenn die beiden Arme getauscht werten (90 Grad Drehung) das
> Signal das mit sich selbst interferiert ist immer das gleiche.

Die Drehung des MMI um 90° ist aber nicht die Phasenverschiebung der 
beiden Lichstrahlen.

Die Phasenverschiebung ergibt sich bei unterschiedlicher Laufzeit.

In deinen Schwingkreisen stellt sich beim Empfänger irgendwann ein 
stationärer Zustand ein (vorausgesetzt, die beiden Frequenzen sind 
gleich).

Wie lange es dauert bis sich dieser stationäre Zustand (in Bezug auf 
Phasenlage und Amplitude) beim Empfänger einstellt, hängt ab von von 
dessen Güte, den Kopplungen, den Amplituden der Sender, deren Frequenz 
und Bandbreite, etc.

Sind die Amplituden der Sender gleich, und verschiebt man die Phase der 
Sender relativ zueinander, dann ändert sich auch die Amplitude am 
Empfänger!

Auf das MMI übertragen führen unterschiedliche Laufzeiten in den Armen 
zu der Phasenverschiebung der eintreffenden Wellen am Beobachtungsort, 
die wiederum eine Änderung der beobachteten Intensität bzw. Lage der 
Streifen bedingt.  Die Streifen selbst entstehen durch minimal 
unterschiedliche Laufzeiten nicht-mittiger Strahlen.  Ändert sich die 
Laufzeitdifferent, so wandern diese Streifen.

von Frank M. (ukw) (Moderator) Benutzerseite


Lesenswert?

Gut erklärt, Johann. Danke.

von Kurt B. (kurt-b)


Lesenswert?

Frank M. schrieb:
> Kurt Bindl schrieb:
>> Es liegt eine Beeinflussung vor, das dürfte doch wohl klar sein.
>
> Deine Rhetorik in Ehren, aber da ist nix klar. In welchem quantivtativen
> Rahmen bewegen sich diese "Beeinflussungen". Kannst Du eine
> Fehlerabschätzung vornehmen?

Es gibt Werte beim Sagnacinterferometer (Herstellerangaben), da wird das 
teileise in % oder Mindestdrehrate angegeben.
Wie gross das ist dass weiss ich jetzt nicht, jedenfalls liegt dieser 
Umstand vor.

Beim MMI sind die erwarteten Phasenunterschiede, die aufgrund der 
vermuteten Laufzeit entstehen, wohl gering, und deshalb gehe ich davon 
aus dass sie durch die Einphaseffekte an den Teilern und Linsen usw. 
noch kleiner werden.
Es werden ja geringe Werte gemessen, das ist aber das was von dem Effekt 
übrigbleibt, die tatsächliche Grösse ist somit unbekannt.


 Kurt

Nachwurf: wer ein MMI zu Hause hat der kann es ja mal probieren, es 
müsste sich beim Einjustieren der Linien/Ringe ein -Einphasverhalten- 
zeigen, so wie es sich auch bei Sendeendstufen zeigt wenn diese nicht 
sauber gegeneinander abgeschottet sind.


.
.

: Bearbeitet durch User
von Kurt B. (kurt-b)


Lesenswert?

Johann L. schrieb:

> Auf das MMI übertragen führen unterschiedliche Laufzeiten in den Armen
> zu der Phasenverschiebung der eintreffenden Wellen am Beobachtungsort,
> die wiederum eine Änderung der beobachteten Intensität bzw. Lage der
> Streifen bedingt.  Die Streifen selbst entstehen durch minimal
> unterschiedliche Laufzeiten nicht-mittiger Strahlen.  Ändert sich die
> Laufzeitdifferent, so wandern diese Streifen.

Nimm einen Kristall und sende von beiden Richtungen zwei Lichtsignale 
knapp nebeneinander durch oder auch nebeneinander mit fast gleicher 
Phasenlage.

Die beiden Signale verändern sich, sie haben nachher eine andere 
Phasenlage als wie wenn nur einer da ist.

Beim MMI bedeutet das das die Laufzeitinformation, die ja ein 
letztendlich eine Phasenänderung ist, verschwindet oder zumindest 
verfälscht wird.

Das wirkt sich dann am Überlagerungsort derart aus dass eben die 
konstruktiven und destruktiven Überlagerungsbereiche nicht da sind wo 
sie wären wenn die Beeinflussung nicht vorhanden wäre.
Sie bleiben, aufgrund der nichterfolgenden Phasenänderung, eben da wo 
sie sind.


 Kurt

Falls es jemanden interessieren sollte:
denkt mal darüber nach was die Ursache der sog. -Verschränkung- ist.
Die beiden Signale werden ebenfalls aus einer gemeinsamen Lichtquelle 
generiert die durch einen Kristall, der durch eine Einwirkquelle 
(geschalteter Laser)in Schwingung versetzt wird.

Was kommt da dann raus, ja, zwei Signale die in ihrer Phasenlage 
zueinander eine feste Beziehung haben.
Auch "verschränkte Signale" genannt.
Das ist das ganze Geheimnis der mysteriösen Verschränkung.

.
.

von U. B. (Gast)


Lesenswert?

> Falls es jemanden interessieren sollte:
> denkt mal darüber nach was die Ursache der sog. -Verschränkung- ist.

Gerne. -

von Kurt B. (kurt-b)


Lesenswert?

U. B. schrieb:
>> Falls es jemanden interessieren sollte:
>> denkt mal darüber nach was die Ursache der sog. -Verschränkung- ist.
>
> Gerne. -

Hm, da hats mich wohl erwischt.

Also, die Polarisation der beiden Lichtpulse liegt beim Verlassen des 
Kristalls vor, ab da sind sie ja unveränderbar.

Die Festlegung geschieht im Kristall und zwar für beide Signale, sie 
sind praktisch miteinander -verschränkt-, also beim Verlassen des 
Kristalls in ihrer Phasenlage zueinander geprägt.

Die Verschränkung funktioniert nur wenn der Kristall in der 
richtigen/passenden Art und Weise ins Schwingen gebracht wird, passt das 
nicht kommt auch keine gegenseitige Abhängigkeit/Korrelation der beiden 
Signale zustande.

Wird nun eine Phasenlage festgestellt ist klar welche die andere hat, 
denn sie wurden ja in Abhängigkeit zueinander erzeugt.

 Kurt

von (prx) A. K. (prx)


Lesenswert?

Kurt Bindl schrieb:
> Weisslichtinterferometrie zeigt, so wie jede Linse es auch macht, dass
> Frequenz- und winkelabhängige Brechung stattfindet

Und mit diesem Effekt kann man streuend reflektierende 3D Oberflächen 
mit Genauigkeit im Nanometerbereich berührungslos abtasten?

http://zygolot.de/library/papers/ATu2A.4.pdf
http://zygolot.de/?/de/met/profilers/

: Bearbeitet durch User
von Robert L. (lrlr)


Lesenswert?

@kurt: schon wieder eine neue Baustelle um vom vorherigen Desaster 
abzulenken?

> also beim Verlassen des
>Kristalls in ihrer Phasenlage zueinander geprägt.

hat diese experiment einen Namen?
hab zu dem Thema nur welche gefunden die KOMPETT anders funktionieren..

: Bearbeitet durch User
von Achim H. (anymouse)


Lesenswert?

Kurt Bindl schrieb:
>> Herrscht denn wenigstens Übereinstimmung darüber, daß das MM-Experiment
>> mit folgenden Theorien vereinbar ist?
>>
>> 1) Emissionstheorie
>> 2) Vollständigge Äthermitführung
>> 3) Bindls Theorie
>> 4) Spezielle Relativitätstheorie
>>
>> Das MM-Experiment ist also nicht in der Lage, zwischen 1), 2), 3) und 4)
>> zu unterscheiden.  Warum wird dann an diesem Experiment geklebt wie
>> Kaugummi?
>
> Weil man in es alles hineininterpretrieren kann was gerade -sinnvoll-
> (nützlich) erscheint.

Nein. Man kann nicht "alles hinein-interpretieren". Allerdings kann 
man "nur" eine Sache aus dem Experiment schließen: Dass die Zeit, welche 
das Licht jeweils für beide Wege (quer und längs der Bewegung) braucht, 
identisch ist, und zwar auch, wenn beide Arme periodisch ihre Rollen 
tauschen.

Deswegen ist das MM-Experiment ein Beweis gegen eine (einfache) 
Theorie des ruhenden Äthers (ohne Annahme einer Längenkontraktion).

Kurt Bindl schrieb:
> es
> müsste sich beim Einjustieren der Linien/Ringe ein -Einphasverhalten-
> zeigen, so wie es sich auch bei Sendeendstufen zeigt wenn diese nicht
> sauber gegeneinander abgeschottet sind.

Wenn es ein Einphasen gibt, so sollte die innerhalb einiger Millionen 
Schwingungen vorbei sein -- bei Licht also Mikrosekunden. Dieses 
Einphasen verhindert aber nicht das auftreten von 
Interferenzerscheinungen, wenn man die Länge eines der Arme verändert, 
selbst nach Stunden.

--

Jetzt zurück zu den gekoppelten Schwingkreisen. Zum ersten müssen alle 
drei Schwingkreise die gleiche Frequenz haben. Ansonsten wird es kaum 
zur Anregung des mittleren Schwingkreises kommen (gleiches gilt für die 
gekoppelten Pendel(uhren) -- bei stark unterschiedlicher Eigenfrequenz).

Du betrachtest zwar die Ergebnisphase, vernachlässigst aber die 
ErgebnisAmplitude. Sind beide Eingangsschwingungen in Phase, so hat die 
Ergebnisschwingung eine (relativ) große Amplitude. Sind die beiden 
Eingangsschwingungen dagegen gegenphasig, so ist die Ergebnisamplitude 
(relativ) sehr klein. Verändert man also die Phase eines der beiden 
Eingangssignale, so wird man eine entsprechende Veränderung der 
Amplitude des Ergebnissignal feststellen.

Das bedeutet, selbst wenn man beim MM-Experiment nur die 
Ergebnisschwingung sehen würde, könnte man trotzdem den sich 
verändernden Phasenunterschied bemerken.

von Achim H. (anymouse)


Lesenswert?

Kurt Bindl schrieb:
> Nimm einen Kristall und sende von beiden Richtungen zwei Lichtsignale
> knapp nebeneinander durch oder auch nebeneinander mit fast gleicher
> Phasenlage.
>
> Die beiden Signale verändern sich, sie haben nachher eine andere
> Phasenlage als wie wenn nur einer da ist.
>
> Beim MMI bedeutet das das die Laufzeitinformation, die ja ein
> letztendlich eine Phasenänderung ist, verschwindet oder zumindest
> verfälscht wird.

Okay, Du hast da aber den Einwand "zwei Lichtsignale ... mit fast 
gleicher Phasenlage".

Was passiert aber, wenn sich die Phasenlage eines der beide Lichtsignale 
vor dem Kristalleintritt ändert?

Zugegeben, auch dann mag die Phasenlage am Ausgang nicht mehr mit der 
Phasenlage am Anfang übereinstimmen. Das ist aber irrelevant. Wichitg 
ist, dass sich das Ergebnis am Ausgang in Abhängigkeit von der 
Phasendifferenz am Eingang des Kristalls ändert.

Aber Du möchtest doch nicht wirklich darüber diskutieren, ob ein 
Michelson-Interferometer geeignet ist, kleine Änderungen der 
Lichtlaufzeit bzw. des Lichtlaufweges zu messen? Dafür sprechen nämlich 
viele viele Anwendungen, die ansonsten alle nicht funktionieren würden.

(Es sei denn, das Michelson-Interferometer würde sich nur bei 
Äther-Experimenten anders verhält als beim Rest).

: Bearbeitet durch User
von Kurt B. (kurt-b)


Lesenswert?

Achim Hensel schrieb:
> Kurt Bindl schrieb:
>> Nimm einen Kristall und sende von beiden Richtungen zwei Lichtsignale
>> knapp nebeneinander durch oder auch nebeneinander mit fast gleicher
>> Phasenlage.
>>
>> Die beiden Signale verändern sich, sie haben nachher eine andere
>> Phasenlage als wie wenn nur einer da ist.
>>
>> Beim MMI bedeutet das das die Laufzeitinformation, die ja ein
>> letztendlich eine Phasenänderung ist, verschwindet oder zumindest
>> verfälscht wird.
>
> Okay, Du hast da aber den Einwand "zwei Lichtsignale ... mit fast
> gleicher Phasenlage".
>

Diese Einschränkung war nicht gewollt gesetzt, eine Beeinflussung findet 
bei jeder Phasenlage statt, sie dürfte nur bei sehr kleiner besonders 
verfälschend ausfallen.


> Was passiert aber, wenn sich die Phasenlage eines der beide Lichtsignale
> vor dem Kristalleintritt ändert?
>

Dann passiert das was passiert, die Phasenlagen die rauskommen sind 
nicht mehr mit denen identisch die vorher waren.
Die beiden Signale haben sich gegenseitig beeinflusst/verändert.
(ich gehe davon aus das sich das besonders bei kleinen Unterschieden 
verändernd auswirkt)


> Zugegeben, auch dann mag die Phasenlage am Ausgang nicht mehr mit der
> Phasenlage am Anfang übereinstimmen. Das ist aber irrelevant. Wichitg
> ist, dass sich das Ergebnis am Ausgang in Abhängigkeit von der
> Phasendifferenz am Eingang des Kristalls ändert.
>

Es ist alles andere als irrrelevant, das was aus den mehrmaligen 
Beeinflussungen rauskommt ist das was verglichen wird, also das Ergebnis 
ausmacht.

Überleg mal, die erste Beeinflussung findet am Strahlteiler statt, die 
zweite ebenfalls und zwar am selben Strahlteiler, dieser bekommt also 
vier Signale zu sehen die sich gegenseitig beeinflussen und das was dann 
gemeinsam zur Aufweitlinse läuft beeinflusst sich dann in dieser 
ebenfalls nochmal.


> Aber Du möchtest doch nicht wirklich darüber diskutieren, ob ein
> Michelson-Interferometer geeignet ist, kleine Änderungen der
> Lichtlaufzeit bzw. des Lichtlaufweges zu messen? Dafür sprechen nämlich
> viele viele Anwendungen, die ansonsten alle nicht funktionieren würden.
>

Ich sage aus dass das was das MMI anzeigt nicht unbedingt mit dem 
übereinstimmt was wirklich anliegt.

Warum sollten denn diese vielen Anwendungen nicht ausreichend 
funktionieren.
Es ist ja nicht so dass da kein Messwert mehr zu erwarten ist, sondern 
dieser halt einen Fehler beinhaltet der auf das Linsenverhalten usw. 
zurückzuführen ist.

Welche Probleme Linsen bei Beobachtung ferner Objekte machen ist ja 
bekannt, die Farbringe, "Sterne" usw. zeigens ja auf.

Bei einer Oberflächenrauheitsmessung fallen Grössenordnungen wie sie 
beim MMI erwarte werden garnicht auf, diese gehen sozusagen unter.

Die Hersteller weisen ja darauf hin dass z.B. ein Sagnacinterferometer 
zur Winkeländerung erst ab einer gewissen Drehrate, also Differenz der 
Laufzeiten in den beiden Lichtwegen, brauchbare Ergebnisse bringt.
Und selbst da muss der -untere Teil- korrigiert werden.


> (Es sei denn, das Michelson-Interferometer würde sich nur bei
> Äther-Experimenten anders verhält als beim Rest).

Warum sollte es das, es gibt keinen Grund dafür.

 Kurt

von Kurt B. (kurt-b)


Lesenswert?

Robert L. schrieb:
> @kurt: schon wieder eine neue Baustelle um vom vorherigen Desaster
> abzulenken?
>
>> also beim Verlassen des
>>Kristalls in ihrer Phasenlage zueinander geprägt.
>
> hat diese experiment einen Namen?
> hab zu dem Thema nur welche gefunden die KOMPETT anders funktionieren..

Die haben doch wohl alle eine Gemeinsamkeit, die beiden Signale kommen 
alle aus einer einzigen Quelle.

 Kurt

von Kurt B. (kurt-b)


Lesenswert?

Achim Hensel schrieb:

>
> Wenn es ein Einphasen gibt, so sollte die innerhalb einiger Millionen
> Schwingungen vorbei sein -- bei Licht also Mikrosekunden. Dieses
> Einphasen verhindert aber nicht das auftreten von
> Interferenzerscheinungen, wenn man die Länge eines der Arme verändert,
> selbst nach Stunden.
>

Das Einphasen ist ja nicht in dem Sinne gemeint dass da eine Schwingung 
ist die dann auf eine andere aufsynchronisiert wird, sondern das sich 
beide auf die jeweils andere "Einphasen", also auf die zubewegen.
Das bedeutet das jede die andere verändert/beeinflusst und das Ergebnis 
nicht mehr der Differenz der beiden Phasenlagen entspricht die sie vor 
der "Einphasung"/Beeinflussung hatten.





> --
>
> Jetzt zurück zu den gekoppelten Schwingkreisen. Zum ersten müssen alle
> drei Schwingkreise die gleiche Frequenz haben. Ansonsten wird es kaum
> zur Anregung des mittleren Schwingkreises kommen (gleiches gilt für die
> gekoppelten Pendel(uhren) -- bei stark unterschiedlicher Eigenfrequenz).
>
> Du betrachtest zwar die Ergebnisphase, vernachlässigst aber die
> ErgebnisAmplitude. Sind beide Eingangsschwingungen in Phase, so hat die
> Ergebnisschwingung eine (relativ) große Amplitude. Sind die beiden
> Eingangsschwingungen dagegen gegenphasig, so ist die Ergebnisamplitude
> (relativ) sehr klein. Verändert man also die Phase eines der beiden
> Eingangssignale, so wird man eine entsprechende Veränderung der
> Amplitude des Ergebnissignal feststellen.
>
> Das bedeutet, selbst wenn man beim MM-Experiment nur die
> Ergebnisschwingung sehen würde, könnte man trotzdem den sich
> verändernden Phasenunterschied bemerken.

Das ist ja alles schön und gut, aber es geht ja nicht um Änderungen die 
in der Gesamthelligkeit sichtbar werden würden, sondern nur in der 
Verschiebung des Ergebnisbildes.

Bei allen MMIs die ich kenne werden solche Überlegungen nicht einbezogen 
und so getan als würde ein MMI zu 100% funktionieren.

Wenn ich überzeugt wäre dass das was ein MMI aussagt, es also perfekt 
funktioniert, auch stimmen würde dann hätte ich einen triftigen Grund 
auf die Bezugsbildung fürs Lichtlaufen auf der Erdoberfläche ohne 
internes aber hinzuweisen.
So aber nicht, denn ich -sehe- dass das MMI, ob alt oder neu, falsch 
misst.

Dass die Masse der Erde das Lichtverhalten beeinflusst ist ja die 
Grundlage für GPS, denn da wird angenommen dass sich das Licht so bewegt 
als wäre der Bezug dafür an die Erde angenagelt.

Ich rechne nicht mit einer 100% tigen Mitnahme/Festlegung, sondern mit 
einer die einen relativ grossen Übergang zum -freiem Raum- hat.
Wie dieser aussieht das hätte ich gerne gewusst.


 Kurt

von Johann L. (gjlayde) Benutzerseite


Lesenswert?

Kurt Bindl schrieb:
> Überleg mal, die erste Beeinflussung findet am Strahlteiler statt, die
> zweite ebenfalls und zwar am selben Strahlteiler, dieser bekommt also
> vier Signale zu sehen die sich gegenseitig beeinflussen und das was dann
> gemeinsam zur Aufweitlinse läuft beeinflusst sich dann in dieser
> ebenfalls nochmal.

Die Strahlen beeinflussen sich nicht gegenseitig; also nicht wie bei 
Effekten der Nichtlinearen Optik.  Was passiert ist störungsfreie 
Überlagerung.

> Die Hersteller weisen ja darauf hin dass z.B. ein Sagnacinterferometer
> zur Winkeländerung erst ab einer gewissen Drehrate, also Differenz der
> Laufzeiten in den beiden Lichtwegen, brauchbare Ergebnisse bringt.

Der Lock-In beim Sagnac-Interferometer kommt durch Rückstreuug an den 
nicht-idealen Spiegeln zustande.  Bei den Vollspiegeln des MMI ist das 
wurscht, denn deren Zweck ist eben genau der, den Strahl *zurück* zu 
spiegeln.  Am Halbspiegel sieht's nicht anders aus:  Wenn's dort 
kleinste Effekte durch Streuung in Strahlrichtung gibt, stört das nicht 
weiter.

Denkbar ist prinzipiell ein zweites, extrem schwaches Interferenzmuster, 
welches das eigentliche überlagert und dadurch zustande kommt, daß Licht 
am Halbspiegel um 180° (!) zurückgestreut wird anstatt im 90°-Winkel 
reflektiert zu werden.  Aber der Effekt ist so schwach daß er 
bestenfalls von theoretischem Interesse ist.

Aber geschenkt, schauen wir uns einfach die Größenordnung des Lock-In 
bei einem üblichen Sagnac-Interferometer an, der mit ca. pi/2 rad/h 
angegeben wird.  Sind wir großzügig und nehmen das 10-fache davon, 
ergeben sich 5pi rad/h ~ 0.005 rad/s.  Abermals überaus großzügig mit 
dem Radius (1m) ergibt eine Geschwindigkeit von 0.005 m/s.

Die Genauigkeit des MMI von 1887 lag grob bei 5 km/s.  Das ist 1/12 des 
erwarteten Wertes wenn es keine Äthermitführung gibt, mithin sollte dies 
bei einer Schwankung von 60 km/s pro Jahr nachweisbar sein.  Der 
hypotetische winzig-Effekt ist 1'000'000 mal kleiner als die 5 km/s, 
geht also komplett unter.

> Welche Probleme Linsen bei Beobachtung ferner Objekte machen ist ja
> bekannt, die Farbringe, "Sterne" usw. zeigens ja auf.

Üblicherweise Beigungsbrgrenzung (hier egal), Kaustik (hier egal), 
chromatische Aberration (hier egal).

Kurt Bindl schrieb:
> Die haben doch wohl alle eine Gemeinsamkeit, die beiden Signale kommen

Also Verschränkung ist schon was anderes als ne einfache Korrelation 
oder Phasenunterschied zweier Wellen.

von Achim H. (anymouse)


Lesenswert?

seufz

Kurt Bindl schrieb:
> Es ist alles andere als irrrelevant, das was aus den mehrmaligen
> Beeinflussungen rauskommt ist das was verglichen wird, also das Ergebnis
> ausmacht.

Nein, eben nicht.

Diese ganzen Beeinflussungen sind entweder statisch (wirken also immer 
gleich und beeinflussen die Phase jedes Lichtbündels), oder aber sie 
haben einen Bezug zur Phasendifferenz.

Kurt Bindl schrieb:
> Das bedeutet das jede die andere verändert/beeinflusst und das Ergebnis
> nicht mehr der Differenz der beiden Phasenlagen entspricht die sie vor
> der "Einphasung"/Beeinflussung hatten.

Es reicht aber, dass das Ergebnis von der Phasendifferenz abhängt. Damit 
ändert sich das Ergebnis, wenn sich die Phasendifferenz ändert.

Und dann ist es egal, ob diese Phasendifferenz durch eine geänderte 
Lichtgeschwindigkeit, Kontraktion des Arms etc. hervorgerufen wird, oder 
aber durch eine manuelle Verschiebung eines der Spiegel.

Kurt Bindl schrieb:
> Bei allen MMIs die ich kenne werden solche Überlegungen nicht einbezogen
> und so getan als würde ein MMI zu 100% funktionieren.

Braucht man auch nicht. Solange man mit einer Änderung des optischen 
Weges aufgrund einer manuellen Spiegelverschiebung eine -Änderung- des 
Interferenzmusters erkennen kann, kann man davon ausgehen, dass die 
gleiche Änderung des optischen Wegs, welche auf einen anderen Effekt 
zurückzuführen ist, ebenfalls die gleiche Interferenzmusteränderung zur 
Folge hat.

Die Mindestdrehzahl beim Sagnacinterferometer bzw. die obere Grenze der 
gemessenen Äther-Relativ-Geschwindigkeit kommen daher, dass diese einer 
nur sehr geringen Änderung der optischen Weg mit einer entsprechenden 
geringen Änderung des Interferenzmusters entsprechen, und so geringe 
Änderungen eben nicht erkannt werden können.


Alle in den obigen Beispieln genannten Effekte können höchstens 
verhindern, dass man bei -jeder- Änderung der optischen Weglänge keine 
Änderung des Interferenzmusters erkennen kann, also auch nicht bei einer 
manuellen Verschiebung eines der Spiegel.


Gesucht wäre also ein Effekt, der bei einer Laufwegsänderung aufgrund 
einer Spiegelverschiebung wirksam wäre, nicht aber bei einer 
Laufwegsänderung aufgrund der unterschiedlichen Orientierung zum relativ 
bewegten Lichtträger.

von Kurt B. (kurt-b)


Lesenswert?

Achim Hensel schrieb:

>
> Gesucht wäre also ein Effekt, der bei einer Laufwegsänderung aufgrund
> einer Spiegelverschiebung wirksam wäre, nicht aber bei einer
> Laufwegsänderung aufgrund der unterschiedlichen Orientierung zum relativ
> bewegten Lichtträger.


Ihr meint also dass ich mir da unnötig Sorgen mache, dass das was da an 
Messergebnis ansteht auch dem entspricht was real ist.
Gut, würde mir eine gewisse Sicherheit bringen.

Wenn die Messwerte stimmen dann ist der Lichtträger an die Erde 
angebunden, wird von ihr sozusagen erzwungen.
Die genau passende Mitführung des sog. Äthers wäre dann ein sehr 
seltsames Geschehen denn warum sollte es ausgerechnet die Erde treffen.
Logischerweise ist anzunehmen dass es Masse ist die da wirkt und den 
LLBezug erzwingt.
Das passt dann zum GPS, zu -Gravitationslinsen-, zur Sternenaberration 
usw.
Heisst: auf dem Mars würde ein GPS-System ebenfalls funktionieren, der 
Fehler durch nicht_einhunder_prozentige Anpassung wäre dann ein 
Indiz/Messwert für die Grösse der Bezugsbildung durch seine Masse.

Gut, aber erst hätte ich gerne Sicherheit in Bezug zum MMI.

 Kurt

: Bearbeitet durch User
von Kurt B. (kurt-b)


Lesenswert?

Johann L. schrieb:
> Kurt Bindl schrieb:
>> Überleg mal, die erste Beeinflussung findet am Strahlteiler statt, die
>> zweite ebenfalls und zwar am selben Strahlteiler, dieser bekommt also
>> vier Signale zu sehen die sich gegenseitig beeinflussen und das was dann
>> gemeinsam zur Aufweitlinse läuft beeinflusst sich dann in dieser
>> ebenfalls nochmal.
>
> Die Strahlen beeinflussen sich nicht gegenseitig; also nicht wie bei
> Effekten der Nichtlinearen Optik.  Was passiert ist störungsfreie
> Überlagerung.
>

Lassen wir das mal so stehen, überzeugt bin ich allerdings nicht.


("Verschränkung")
> Kurt Bindl schrieb:
>> Die haben doch wohl alle eine Gemeinsamkeit, die beiden Signale kommen
>
> Also Verschränkung ist schon was anderes als ne einfache Korrelation
> oder Phasenunterschied zweier Wellen.

Klar, es geht um die Erzeugung der Polarität der beiden Signale.
Und soweit ich aus dem Büchlein von Zeilinger entnommen habe ist da eine 
fieslige Einstellung notwendig damit das Ergebnis passt, also die 
Polarisationen der beiden Signale zueinander OK/brauchbar ist.
Das wird dadurch erreicht das sie eine gemeinsame Quelle haben und ihre 
Polarisationsänderung durch einen Vorgang gemeinsam stattfindet.

Sie sind also fertig wenn sie den Kristall verlassen und das wars auch 
schon.
Jede weitere Hineininterpretation von ominöser Fernwirkung ist 
Wunschdenken.

 Kurt

von (prx) A. K. (prx)


Lesenswert?

Kurt Bindl schrieb:
> Ihr meint also dass ich mir da unnötig Sorgen mache, dass das was da an
> Messergebnis ansteht auch dem entspricht was real ist.

Könnte man so sagen. Denn wenn du um die reale Funktionsfähigkeit dieser 
Interferometrie herum kommen willst, müsstest du schon auf "Die Realität 
existiert nicht!" ausweichen. Es gibt ein Bisschen zu viele erfolgreich 
genutzte Anwendungen davon.

> Logischerweise ist anzunehmen dass es Masse ist die da wirkt und den
> LLBezug erzwingt.

Damit bist du circa hier angekommen: 
https://de.wikisource.org/wiki/Translatorische_Bewegung_des_Lichtäthers

Beim Versuch, M&M und Äthertheorie in Einklang zu bringen, landete man 
letztlich bei der Lorentzschen Äthertheorie. Auf die dann die Undenkbare 
folgte.

: Bearbeitet durch User
von Kurt B. (kurt-b)


Lesenswert?

A. K. schrieb:
> Kurt Bindl schrieb:
>
>> Logischerweise ist anzunehmen dass es Masse ist die da wirkt und den
>> LLBezug erzwingt.
>
> Damit bist du circa hier angekommen:
> https://de.wikisource.org/wiki/Translatorische_Bewegung_des_Lichtäthers
>
> Beim Versuch, M&M und Äthertheorie in Einklang zu bringen, landete man
> letztlich bei der Lorentzschen Äthertheorie. Auf die dann die Undenkbare
> folgte. Bloss bist du ja mit Lorentz auch schon über Kreuz.

Genau da will ich nicht hin
Auszug:
"Die Neigung, die Eigenschaften des Aethers mit denen der ponderabeln 
Materie in Uebereinstimmung zu bringen, hat zu der Annahme geführt, der 
Aether könne Bewegungen nach Art einer Flüssigkeit ausführen"

Für mich ist dieses -Ding- ein Extremes, extrem dicht und starr (im 
Nahbereich zumindest) und alles was sich bewegen wollte würde sofort 
darin erstarren.
Also nix mit superflüssig oder sowas ähnliches.

Wie Bewegung von Materie darin möglich ist hab ich ja schon angedacht, 
wies mit Licht sein könnte ebenfalls.

Obs so ist, ich weiss es jedenfalls nicht (hoffe halt einfach).

 Kurt

(es muss alles zusammenpassen, sonst taugts nichts)

von Johann L. (gjlayde) Benutzerseite


Lesenswert?

Kurt Bindl schrieb:
> Wenn die Messwerte stimmen dann ist der Lichtträger an die Erde
> angebunden, wird von ihr sozusagen erzwungen.
> Die genau passende Mitführung des sog. Äthers wäre dann ein sehr
> seltsames Geschehen denn warum sollte es ausgerechnet die Erde treffen.
> Logischerweise ist anzunehmen dass es Masse ist die da wirkt und den
> LLBezug erzwingt.
> Das passt dann zum GPS, zu -Gravitationslinsen-, zur Sternenaberration
> usw.

Wieso passt das zur Aberration? Die sollte dann doch eben *nicht* 
vorhanden sein.

von (prx) A. K. (prx)


Lesenswert?

Kurt Bindl schrieb:
> Genau da will ich nicht hin

Deshalb "circa". Die fussen alle auf Maxwell und dessen 
Transversalwellen behagen dir ja auch schon nicht.

von Kurt B. (kurt-b)


Lesenswert?

Johann L. schrieb:
> Kurt Bindl schrieb:
>> Wenn die Messwerte stimmen dann ist der Lichtträger an die Erde
>> angebunden, wird von ihr sozusagen erzwungen.
>> Die genau passende Mitführung des sog. Äthers wäre dann ein sehr
>> seltsames Geschehen denn warum sollte es ausgerechnet die Erde treffen.
>> Logischerweise ist anzunehmen dass es Masse ist die da wirkt und den
>> LLBezug erzwingt.
>> Das passt dann zum GPS, zu -Gravitationslinsen-, zur Sternenaberration
>> usw.
>
> Wieso passt das zur Aberration? Die sollte dann doch eben *nicht*
> vorhanden sein.

Von Aussen betrachtet verhält sich der LlBez der Erde wie ein Strudel 
(wegen ihrer Rotation), es sind also gekrümmte Lichtwege vorhanden.
Licht das darin läuft wird also gebogen, das ist das was dann als 
-Ortsveränderung- des Sterns sichtbar wird.

An den Polen sollte das gering sein, am Äquator am stärksten.


 Kurt

von Frank M. (ukw) (Moderator) Benutzerseite


Lesenswert?

Kurt Bindl schrieb:
> Von Aussen betrachtet verhält sich der LlBez der Erde wie ein Strudel
> (wegen ihrer Rotation), es sind also gekrümmte Lichtwege vorhanden.
> Licht das darin läuft wird also gebogen, das ist das was dann als
> -Ortsveränderung- des Sterns sichtbar wird.
>
> An den Polen sollte das gering sein, am Äquator am stärksten.

Du willst damit sagen, dass die Rotation der Erde (und nicht die Masse) 
eine Krümmung des Lichts verursacht?

Kannst Du da ein Messergebnis vorlegen, was das bestätigt?

von Kurt B. (kurt-b)


Lesenswert?

Frank M. schrieb:
> Kurt Bindl schrieb:
>> Von Aussen betrachtet verhält sich der LlBez der Erde wie ein Strudel
>> (wegen ihrer Rotation), es sind also gekrümmte Lichtwege vorhanden.
>> Licht das darin läuft wird also gebogen, das ist das was dann als
>> -Ortsveränderung- des Sterns sichtbar wird.
>>
>> An den Polen sollte das gering sein, am Äquator am stärksten.
>
> Du willst damit sagen, dass die Rotation der Erde (und nicht die Masse)
> eine Krümmung des Lichts verursacht?
>

Die Masse der der Erde erzwingt ihren LlBez, Licht in ihrer Nähe läuft 
so als sei das Medium dazu an die Erde angenagelt.
Die Bindung wird nach aussen hin immer geringer, der LlBez immer 
schwächer und es dominiert dann der der von der Sonne erbracht wird, 
dann der des Galaxiearms, der Galaxie.... bis zum Bereich den das 
Universum da lokal zur Verfügung stellt.

Es ist das Wirken der einzelnen Masseteilchen, der BT also, die das 
anstellen.
Die Masse erzwingt den LlBezug, da die Masse der Erde rotiert rotiert 
der natürlich (mehr oder weniger) mit, und das ergibt für Licht das von 
aussen kommt letztendlich eben einen krummen Weg.
Das ist für mich die Ursache der beobachtbaren Sternenaberration.


> Kannst Du da ein Messergebnis vorlegen, was das bestätigt?

Hast du viell. Fragen die nicht so utopische sind (woher sollte ich da 
denn Messergebnisse haben).  :-)



 Kurt

von Frank M. (ukw) (Moderator) Benutzerseite


Lesenswert?

Kurt Bindl schrieb:
> Die Masse erzwingt den LlBezug, da die Masse der Erde rotiert rotiert
> der natürlich (mehr oder weniger) mit, und das ergibt für Licht das von
> aussen kommt letztendlich eben einen krummen Weg.

Nochmal, weil Du schon wieder schwafelnd ausweichst:

Die Krümmung durch Licht wird also nicht durch die Masse, sondern 
ausschließlich durch die Rotation der Erde verursacht?

> Das ist für mich die Ursache der beobachtbaren Sternenaberration.

Dann lies mal:

  http://de.wikipedia.org/wiki/Aberration_%28Astronomie%29

Der Artikel endet damit, dass die Aberration des Licht viel einfacher 
durch die SRT erklärt werden kann.

Die entsprechende Aberrationsformel zur Bestimmung der relativen Winkel 
in zueinander bewegten Inertialsystemen kann man hier einfach durch 
Anwendung der relativistischen Geschwindigkeitsaddition ableiten. So 
simpel ist das.

Da muss man keinen "Strudel" oder einen an die Erde "angetackerten 
Träger" erfinden, um das zu verstehen.

>> Kannst Du da ein Messergebnis vorlegen, was das bestätigt?
>
> Hast du viell. Fragen die nicht so utopische sind (woher sollte ich da
> denn Messergebnisse haben).  :-)

Komisch, andere haben das schon 1725 hinbekommen. Schwach, Kurt. Echt 
schwach.

von (prx) A. K. (prx)


Lesenswert?

Frank M. schrieb:
> Der Artikel endet damit, dass die Aberration des Licht viel einfacher
> durch die SRT erklärt werden kann.

Occams Razor und "einfacher" sind etwas subjektiv. Sein Problem ist doch 
grad, dass die SRT fundamental inkompatibel mit seiner Vorstellung und 
deshalb für ihn nicht einfacher als nichtrelativistische Theorien ist. 
Somit ist das für ihn kein Argument.

Eine Hypothese ist nicht a priori richtig oder falsch, nur weil sie 
einfacher oder komplexer wirkt. Das ist der nachvollziehbare Kern seiner 
Argumentation über Vorurteilsfreiheit. Was natürlich nicht heisst, dass 
er selber vorurteilsfrei vorginge.

Allerdings schwant mir, dass er den Ärger mit M&M durch den Strudel 
nicht so einfach los wird, denn dann wird es wohl etwas kompliziert, M&M 
und andere Experimente zur Lichtausbreitung von der exakten Lage und 
Position der Messebene unabhängig zu machen.

: Bearbeitet durch User
von Kurt B. (kurt-b)


Lesenswert?

Frank M. schrieb:
> Kurt Bindl schrieb:
>> Die Masse erzwingt den LlBezug, da die Masse der Erde rotiert rotiert
>> der natürlich (mehr oder weniger) mit, und das ergibt für Licht das von
>> aussen kommt letztendlich eben einen krummen Weg.
>
> Nochmal, weil Du schon wieder schwafelnd ausweichst:
>
> Die Krümmung durch Licht wird also nicht durch die Masse, sondern
> ausschließlich durch die Rotation der Erde verursacht?
>

Ein Teil der Sternenaberration ergibt sich aus Sicht der Erdoberfläche 
wenn senkrecht nach oben geschaut wird.
Da läuft das Signal, aufgrund der sich drehenden Erde und dem dabei sich 
mitdrehendem Bezug, eine krumme Bahn, je näher an der Oberfläche desto 
krummer.

Das heisst nicht das die Rotation schuld an der Krümmung des Lichtweges 
ist.
Bei Galaxien ist die Rotation und das dabei auftretende Verhalten von 
Materie zu erkennen.
Es scheint so als wurde im Mittelpunkt der Galaxie der -Stopsel- 
gezogen, dass darin irgendwas verschwindet das dann von aussen nachkommt 
(so wie der Studel in der Badewanne.

Meine Version ist dass Materie den "Trägerdruck" abbaut, dieser somit im 
Zentrum geringer ist und darum von aussen her ausgeglichen wird.
Es ergibt sich also ein quasi Ausgleichsstrom (Druckausgleich (ev. auch 
Substanznachschub?)) der nach innen drückt und die Rotation 
ankurbelt/erhält.
Licht das da läuft folgt diesem -Ausgleichsstrom- und wird dadurch 
abgelenkt.
Diese Ablenkung ist zu sehen wenn Licht an grossen Massen vorbeizieht, 
auch als Gravitationslinse bekannt.


>> Das ist für mich die Ursache der beobachtbaren Sternenaberration.
>
> Dann lies mal:
>
>   http://de.wikipedia.org/wiki/Aberration_%28Astronomie%29
>
> Der Artikel endet damit, dass die Aberration des Licht viel einfacher
> durch die SRT erklärt werden kann.
>

Du meinst ich soll Vertrauen in die RT haben?
Wie denn?
- es werden Wellen behauptet die nicht sind (Transversalwellen bei 
Licht)
- es wird ein Doppler behauptet der nicht ist, Transversaldoppler
- es wird Lichtinvarianz behauptet die nicht existiert
- es werden relativistische Effekte behauptet die nicht existieren
- es müssen sich Längen verkürzen damits zu Postulaten passt.
- es müssen Begrenzungen eingeführt werden und dann über Winkeladdition 
von Geschwindgikeitsangaben (die wiederum keinen Bezug haben sondern nur 
eine Zahl) kleingerechnet werden

Naja, mag glauben wer was davon hat, ich muss das nicht.


> Die entsprechende Aberrationsformel zur Bestimmung der relativen Winkel
> in zueinander bewegten Inertialsystemen kann man hier einfach durch
> Anwendung der relativistischen Geschwindigkeitsaddition ableiten. So
> simpel ist das.
>

Es gibt kein IS, eins anzunehmen ist ja OK, aber man sollte sich bewusst 
sein was damit angerichtet wird.

Diesem IS wird ein Absolutbezug zugewiesen der in der Realität nicht 
existiert, somit sind die Aussagen, wie z.B Geschwindgkeit usw. auf 
einen nichtexistierenden, sondern postulierten Bezug aufgesetzt.
Das hat nichts mehr mit der Realität zu tun.


 Kurt

von Frank M. (ukw) (Moderator) Benutzerseite


Lesenswert?

Kurt Bindl schrieb:
> - es werden Wellen behauptet die nicht sind (Transversalwellen bei
> Licht)

Dann erklär mir bitte, wie ein Polarisationsfilter vor einer Kamera 
funktioniert.

  http://de.wikipedia.org/wiki/Polarisationsfilter

Dieser nutzt nämlich die Eigenschaft aus, dass Lichtwellen transversal 
sind.

Genau bei dieser Frage waren wir schon vor ca. 2 Jahren, Kurt. Du kannst 
sie nicht beantworten. Und damit fällt bei Dir alles zusammen wie ein 
Kartenhaus.

von Frank M. (ukw) (Moderator) Benutzerseite


Lesenswert?

A. K. schrieb:
> Occams Razor und "einfacher" sind etwas subjektiv. Sein Problem ist doch
> grad, dass die SRT fundamental inkompatibel mit seiner Vorstellung und
> deshalb für ihn nicht einfacher als nichtrelativistische Theorien ist.
> Somit ist das für ihn kein Argument.

Für ihn nicht. Aber ohne Ockhams Rasiermesser wären wir heute noch in 
der Steinzeit, denn man kann natürlich alles komplzierter machen als es 
tatsächlich ist.

> Allerdings schwant mir, dass er den Ärger mit M&M durch den Strudel
> nicht so einfach los wird, denn dann wird es wohl etwas kompliziert, M&M
> und andere Experimente zur Lichtausbreitung von der exakten Lage und
> Position der Messebene unabhängig zu machen.

Genau das "Komplizierte" an Kurts Gehabe führt notgedrungen irgendwann 
zu Widersprüchen. Und genau da kommt er nicht mehr raus. Er behandelt 
Schall und Licht gleich, obwohl sich es bei Schall um longitudinale 
Wellen, bei Licht um Transversalwellen handelt.

Ein simples Polarisationsfilter vor einer handelsüblichen Kamera, 
welches Reflektionen von Glas oder Wasseroberflächen wegfiltert, bricht 
ihm das Genick. Da kann er sich winden und wenden, wie er will.

von Frank M. (ukw) (Moderator) Benutzerseite


Lesenswert?

Kurt Bindl schrieb:
> Diese Ablenkung ist zu sehen wenn Licht an grossen Massen vorbeizieht,
> auch als Gravitationslinse bekannt.

Ganz genau. Nur ist diese Ablenkung komplett unabhängig von der Rotation 
der Masse. Es zählt ausschließlich die Masse. Nix Strudel, einfach ART 
in ihrer schönsten Form.

Dumm gelaufen, Kurt.

: Bearbeitet durch Moderator
von (prx) A. K. (prx)


Lesenswert?

Sein Strudel muss sowieso beides mitnehmen, Rotation und Masse, oder 
eben rotierende Masse. Für die tägliche Aberration braucht er die 
Rotation. Aber wo ist die bei der jährlichen Aberration, also aufgrund 
Erde um Sonne?

von (prx) A. K. (prx)


Lesenswert?

Frank M. schrieb:
> Für ihn nicht. Aber ohne Ockhams Rasiermesser wären wir heute noch in
> der Steinzeit, denn man kann natürlich alles komplzierter machen als es
> tatsächlich ist.

Kleine Anekdote zur Psychologie der Akzeptanz der Welt der RT. Lorentz, 
wahrlich kein unbeschriebenes Blatt in der Frage, hat die RT (und 
Einstein) sehr wohl aktiv unterstützt, ist aber trotzdem zeitlebens nie 
vom Äther weggekommen.

Einstein über Lorentz: "Dass er diesen Schritt zur speziellen 
Relativitätstheorie nicht machte lag einfach darin, dass es für ihn 
psychologisch unmöglich war, auf die Realität des Äthers als eines 
materiellen Dinges (Trägers des elektromagnetischen Feldes) zu 
verzichten. Wer diese Zeit miterlebt hat, begreift es."

von Kurt B. (kurt-b)


Lesenswert?

Frank M. schrieb:
> Kurt Bindl schrieb:
>> - es werden Wellen behauptet die nicht sind (Transversalwellen bei
>> Licht)
>
> Dann erklär mir bitte, wie ein Polarisationsfilter vor einer Kamera
> funktioniert.
>
>   http://de.wikipedia.org/wiki/Polarisationsfilter
>
> Dieser nutzt nämlich die Eigenschaft aus, dass Lichtwellen transversal
> sind.
>
> Genau bei dieser Frage waren wir schon vor ca. 2 Jahren, Kurt. Du kannst
> sie nicht beantworten. Und damit fällt bei Dir alles zusammen wie ein
> Kartenhaus.

Meinst du, ich mein das nicht.
Wir müssen nur ein bisserl tiefer einsteigen, aber wir waren doch schon 
bei den beiden sich ausbreitenden Druckzuständen die beim Dipol 
entstehen, und auch beim Empfangsdipol, da wo man sieht wieso dieser 
eine Spannung erzeugt, hier können wir dann aufsetzen.

Als erstes schauen wir uns an wie so ein Polfilter aufgebaut ist und 
funktioniert.

Es sind lauter Längsstreifen von Material das mit Licht in Resonanz 
gehen kann und das zugleich eine bestimmte Anordnung seiner einzelnen 
Resonanzkörper aufweist.
Das ist nämlich Voraussetzung fürs Polfilter.

Liegen die einzelnen Resonanzkörper durcheinander, geht nichts durchs 
Filter durch, sind sie geordnet dann wohl, aber nur wenn die Anordnung 
einer bestimmten Muster folgt.

 Kurt

von Kurt B. (kurt-b)


Lesenswert?

Frank M. schrieb:
> Kurt Bindl schrieb:
>> Diese Ablenkung ist zu sehen wenn Licht an grossen Massen vorbeizieht,
>> auch als Gravitationslinse bekannt.
>
> Ganz genau. Nur ist diese Ablenkung komplett unabhängig von der Rotation
> der Masse. Es zählt ausschließlich die Masse. Nix Strudel, einfach ART
> in ihrer schönsten Form.
>
> Dumm gelaufen, Kurt.

Tja, wenn sich keine -tägliche- Aberration zeigt, wenn also am Pol und 
am Äquator kein Unterschied besteht der durch die Rotation der Erde 
verursacht wird dann gibt's zwei Möglichkeiten:

a'
Die Erde erzeugt keinen eigenen LlBez und es gilt das "zwängt sich durch
den "Äther" durch, dann funktioniert auch GPS nicht.


b'
der LlBez ist do "dünn" dass es nicht auffällt.

und c' es hat noch niemand zwischen Pol und Äquator beim Hinschauen 
unterschieden.

 Kurt

von Johann L. (gjlayde) Benutzerseite


Lesenswert?

Kurt Bindl schrieb:
> Von Aussen betrachtet verhält sich der LlBez der Erde wie ein Strudel
> (wegen ihrer Rotation), es sind also gekrümmte Lichtwege vorhanden.
> Licht das darin läuft wird also gebogen, das ist das was dann als
> -Ortsveränderung- des Sterns sichtbar wird.
>
> An den Polen sollte das gering sein, am Äquator am stärksten.

Ist aber nicht so ;-)

Bei Bewegung mit Geschwindigkeit v senkrecht zur Beobachtungsrichtung 
ist die Größe der Aberration v/c in Richtung der Bewegung.  Die 
Erdrotation hat zwar Anzeil daran (am Äquator ca 500 m/s) ist aber 
wesentlich kleiner als der von der Bewegung der Erde um die Sonne (30 
km/s).  Pro Halbjahr ist das eine Aberration von 0.7' und bissl 
Kleinkram wegen Erdrotation (weniger als 1/50 davon).

Wenn dein Bezug das Licht mitführt, also Bewegung von Masse ihn 
"mitreisst" dann sollte die Aberration kleiner ausfallen als sie es tut 
und an den Polen fast ganz verschwinden, was nicht der Fall ist.

Außerdem hat der Bezug das Problem, daß er dann auch von der Sonne 
mitgeführt wird.  An der Seite der Sonne, die sich von uns weg dreht, 
wäre die Lichtgeschwindigkeit etwas kleiner, was daher zu einer 
Ablenkung zur Sonne hin führen würde. Auf der anderen Seite der Sonne 
jedoch, die zu uns hin rotiert, wäre die Lichtgeschwindigkeit etwas 
größer, und damit würde das Licht von der Sonne weg abgelenkt!

Im Endeffekt kommt dein Bezug darauf raus, wo auch hydrodynamische 
Modelle des Äthers gelandet sind...

: Bearbeitet durch User
von (prx) A. K. (prx)


Lesenswert?

Kurt Bindl schrieb:
> und c' es hat noch niemand zwischen Pol und Äquator beim Hinschauen
> unterschieden.

Die Aberration durch Erdrotation liegt abhängig von der Breite bei 
maximal 0,32 Bogensekunden. Für Hobbyastronomen ist das folglich 
irrelevant.

Die Präzision von astronomischer Positionsbestimmung per VLBI liegt 
jedoch bei unter einer Bogenmillisekunde. Abweichungen vom theoretischen 
Wert wären unweigerlich aufgefallen.

: Bearbeitet durch User
von Kurt B. (kurt-b)


Lesenswert?

Johann L. schrieb:
> Kurt Bindl schrieb:
>> Von Aussen betrachtet verhält sich der LlBez der Erde wie ein Strudel
>> (wegen ihrer Rotation), es sind also gekrümmte Lichtwege vorhanden.
>> Licht das darin läuft wird also gebogen, das ist das was dann als
>> -Ortsveränderung- des Sterns sichtbar wird.
>>
>> An den Polen sollte das gering sein, am Äquator am stärksten.
>
> Ist aber nicht so ;-)

Was du einfach mal so annimmst.


>
> Bei Bewegung mit Geschwindigkeit v senkrecht zur Beobachtungsrichtung
> ist die Größe der Aberration v/c in Richtung der Bewegung.  Die
> Erdrotation hat zwar Anzeil daran (am Äquator ca 500 m/s) ist aber
> wesentlich kleiner als der von der Bewegung der Erde um die Sonne (30
> km/s).  Pro Halbjahr ist das eine Aberration von 0.7' und bissl
> Kleinkram wegen Erdrotation (weniger als 1/50 davon).
>

Und dieser Kleinkram der 500 m/s würde aufzeigen ob da ein Unterscheid 
zu den Polen besteht.

Angenommen der LlBez reicht 10000 Km in den Raum hinein dann würde der 
erste Teil überhaupt nicht rotieren, die Teile bis zur Oberfläche immer 
mehr mitgehen.
Das bedeutet dass nur Teil der Laufzeit durch die 10000 Km beiträgt 
überhaupt eine seitliche Ablenkung aufzubauen.


> Wenn dein Bezug das Licht mitführt, also Bewegung von Masse ihn
> "mitreisst" dann sollte die Aberration kleiner ausfallen als sie es tut
> und an den Polen fast ganz verschwinden, was nicht der Fall ist.
>

Ich sehe nur einen Unterschied zwischen Pol und Äquator, also eine 
Grundaberration, erhöht durch die Bewegung am Äquator.


> Außerdem hat der Bezug das Problem, daß er dann auch von der Sonne
> mitgeführt wird.  An der Seite der Sonne, die sich von uns weg dreht,
> wäre die Lichtgeschwindigkeit etwas kleiner, was daher zu einer
> Ablenkung zur Sonne hin führen würde. Auf der anderen Seite der Sonne
> jedoch, die zu uns hin rotiert, wäre die Lichtgeschwindigkeit etwas
> größer, und damit würde das Licht von der Sonne weg abgelenkt!
>

Ist es nicht so dass Sterne einen -anderen Ort- zeigen wenn sie hinter 
oder neben der Sonne sind?
Ich das die Plasmaeinwirkung der Sonnenoberfläche oder eine 
-Gravitationslinse-.

Hat schon mal jemand eine Sternenaberration beobachtet wenn die Sonne 
direkt oder sehr nahe neben der Quelle steht?


> Im Endeffekt kommt dein Bezug darauf raus, wo auch hydrodynamische
> Modelle des Äthers gelandet sind...

In Endeffekt ist es egal was rauskommt, es sollte nur möglichst gut die 
Realität widerspiegeln.
Und diese ist ein verdammt scheues Reh und kennt allerlei Tricks um sich 
zu verbergen, besonders diese: sich in Sicherheit zu wiegen.

 Kurt

(bisher sind alle "Wahrheiten" in Bezug zur Natur den Bach 
runtergegangen, ich mache mir keine Hoffnung dass das zu Ende sein 
sollte)

von (prx) A. K. (prx)


Lesenswert?

Kurt Bindl schrieb:
> Hat schon mal jemand eine Sternenaberration beobachtet wenn die Sonne
> direkt oder sehr nahe neben der Quelle steht?

Bitteschön: http://arxiv.org/pdf/gr-qc/0205061

Das Papier beschreibt zwar nicht die Beobachtung selbst, sondern die 
Faktoren, die darin eingehen, aber angesichts des Umstands, dass man 
dies als Grundlage echter Beobachtung durch reale Instrumente verwendet 
und auf vorherigen Erfahrungen fusst, macht das keinen grossen 
Unterschied.

Weniger gefallen wird dir, was man da alles berücksichtigen muss.

Was glaubst du eigentlich, was die Astronomen so machen? Bibelstudien?

: Bearbeitet durch User
von Kurt B. (kurt-b)


Lesenswert?

A. K. schrieb:
> Kurt Bindl schrieb:
>> Hat schon mal jemand eine Sternenaberration beobachtet wenn die Sonne
>> direkt oder sehr nahe neben der Quelle steht?
>
> Bitteschön: http://arxiv.org/pdf/gr-qc/0205061

Danke.

>
> Das Papier beschreibt zwar nicht die Beobachtung selbst,

Genau das hätte ich mir gewünscht, die Frage: wurde am Äquator und am 
Pol gemessen ist wohl hinfällig, denn ich sehe dass da Punktmassen 
angenommen werden.


< sondern die
> Faktoren, die darin eingehen, aber angesichts des Umstands, dass man
> dies als Grundlage echter Beobachtung durch reale Instrumente verwendet
> und auf vorherigen Erfahrungen fusst, macht das keinen grossen
> Unterschied.
>

Ich deute natürlich die Aussagen zur -Gravitation bewirkt- um in: der 
LlBez bewirkt, das macht den Unterschied da es
a' keine Gravitation gibt die irgendwas bewirkt weil sie selber ein 
Sekundärprodukt der Ortsumstände ist und
b' einen Träger für Licht gibt denn ohne diesen gibt's keine gesetzten 
Geschwindigkeiten und keinerlei Ordnung und kein Wellenverhalten.
Keine Welle ohne Träger, egal ob bei Schall oder bei Licht.

Was mich immer noch unzufrieden sein lässt ist der Umstand dass ich 
nicht erkennen kann ob der Bezug fürs Lichtlaufen und der Bezug für 
Materie ein und derselbe ist.

Das löst sich auf wenn ich sicher sei kann dass das MMI funktioniert und 
der Laserkreisel auch, den dann sind zwei aussagekräftige Messgeräte 
vorhanden die eindeutig aufzeigen dass es zwei Bezüge sind.


> Weniger gefallen wird dir, was man da alles berücksichtigen muss.
>

Ob mir das gefällt ist eigentlich egal, ich bin für jede Information 
dankbar.

> Was glaubst du eigentlich, was die Astronomen so machen? Bibelstudien?

Sie sind unsere Zukunft denn die liegt wohl nicht unbedingt auf der 
Erde.


 Kurt

(damit das klar ist, ich habe höchsten Respekt vor Leuten die 
Wissenschaft betreiben, unabhängige Wissenschaft, mir "kraust" vor 
Leuten die auf glaubensähnlicher Verhaltensweise agieren und meisst nur 
in der Lage sind Links zu präsentieren wo das was bewiesen werden soll 
angeblich drinsteht ohne selber die gemachten Zusammenhänge aufzeigen zu 
können)

von (prx) A. K. (prx)


Lesenswert?

Kurt Bindl schrieb:
> Genau das hätte ich mir gewünscht, die Frage: wurde am Äquator und am
> Pol gemessen ist wohl hinfällig, denn ich sehe dass da Punktmassen
> angenommen werden.

Wäre auch nicht so leicht zu beantworten gewesen. Also wo bei dem 
Astronomiesatelliten, um den es dabei ging, Äquator und Pol sind ;-). 
SIM wurde zwar gecancelled, aber der letztes Jahr gestartete Gaia ist 
ähnlich. Und sitzt in einem Lagrange-Punkt.

> a' keine Gravitation gibt die irgendwas bewirkt weil sie selber ein
> Sekundärprodukt der Ortsumstände ist und

Beim Vorgänger Hipparcos stellte man bei der Datenauswertung fest, dass 
eine Korrektur aufgrund Gravitionseinfluss erforderlich ist. Und 
Hipparcos war auf 1mas genau, Gaia ist es auf 25µas - deshalb dieses 
Paper über die vielen Einflüsse.

: Bearbeitet durch User
von (prx) A. K. (prx)


Lesenswert?

Kurt Bindl schrieb:
> (damit das klar ist, ich habe höchsten Respekt vor Leuten die
> Wissenschaft betreiben, unabhängige Wissenschaft,

Wieso gehst du dann davon aus, dass die Physiker der letzten 200 Jahre 
hauptsächlich irgendwelche hypothetischen Modelle ausbrüten, ohne sie 
jemals zu überprüfen? Den ganzen Thread durchziehen Vorwürfe von dir, 
dass von dir bezweifelte oder negierte Aussagen wohl nie überprüft 
worden seien.

Wieso gehst du dann oft davon aus, dass Physiker, die dann doch 
irgendwas ausprobieren, von ihrem Fach absolut keine Ahnung haben und 
kindische Fehler machen? Und auch jene Kollegen und Konkurrenten, denen 
das Ergebnis absolut nicht ins Konzept passt, solche einfachen Fehler 
nicht sehen. Aber du.

Das sind eben keine Religionsschüler, die die ganze Zeit tradierte Texte 
des Heiligen Einstein und die sakrosankten Interpretationen lernen.

Oder ist Unabhängigkeit nur dort gegeben, wo dir das Ergebnis passt?

: Bearbeitet durch User
von (prx) A. K. (prx)


Lesenswert?

Kurt Bindl schrieb:
> Genau das hätte ich mir gewünscht, die Frage: wurde am Äquator und am
> Pol gemessen ist wohl hinfällig, denn ich sehe dass da Punktmassen
> angenommen werden.

Und falls du die Himmelskörper meinen solltest, deren Einflüsse 
kompensiert werden müssen: Tu dir keinen Zwang an. Gaias Ergebnisse 
werden vorraussichtlich 2022 vorliegen, und bei den Millionen Sternen, 
deren Position vermessen werden dürfte, ist garantiert alles dabei.

Die Daten der ca. 120000 Sterne von Hipparcos sind auch öffentlich. Und 
die Paper zu dieser Mission gibts in der Bibliothek oder 
Fachbereichsbibliothek der nächsten passenden Uni.

: Bearbeitet durch User
von Johann L. (gjlayde) Benutzerseite


Lesenswert?

A. K. schrieb:
> Wieso gehst du dann oft davon aus, dass Physiker, die dann doch
> irgendwas ausprobieren, von ihrem Fach absolut keine Ahnung haben und
> kindische Fehler machen? Und auch jene Kollegen und Konkurrenten, denen
> das Ergebnis absolut nicht ins Konzept passt, solche einfachen Fehler
> nicht sehen. Aber du.

Dunning-Kruger-Effekt?

von (prx) A. K. (prx)


Lesenswert?

Kurt Bindl schrieb:
> Das löst sich auf wenn ich sicher sei kann dass das MMI funktioniert

Ich dachte, dieses Thema hätten wir durch. Durcher gehts nicht, da 
solche oder damit eng verwandte Geräte überall im Einsatz sind. Wenn dir 
das nicht als Funktionsbeweis reicht, dann sind wir endlich beim 
letzthin erwähnten "Die Realität gibt es nicht!" angekommen.

: Bearbeitet durch User
von (prx) A. K. (prx)


Lesenswert?

Kurt Bindl schrieb:
> Genau das hätte ich mir gewünscht, die Frage: wurde am Äquator und am
> Pol gemessen

Hatte ich vorhin schon angeführt. Die VLBI berücksichtigt für 
Kompensation der Aberration durch Erdrotation den Breitengrad. Folglich 
ist diese Frage abschliessend geklärt.

von Johann L. (gjlayde) Benutzerseite


Lesenswert?

Für die jährliche Aberration ist egal, von wo aus auf der Erde 
gemessen wird; entscheidend ist die Richtung:  Senkrecht zur Bahn der 
Erde um die Sonne.

Die tägliche Aberration ist maximal in der durch Beobachtungsort und 
Erdachse gegebenen Ebene und abhängig von der geographischen Breite. 
(Steht ein Stern am Äquator im Zenit, dann hat man zwar die maximale 
tägliche Aberration; allerdings steht der Stern 1/2 Tag später im Nadir 
und kann deshalb nicht beobachtet werden.)

Insgesamt ergibt sich die Aberration aus jährlicher und täglicher 
Aberration, wobei letztere ca. um den Faktor 60 kleiner ist.

Die Aberration wurde eher zufällig entdeckt, nämlich im 18. Jahrhundert 
auf der Suche nach der Fixstern-Parallaxe.  Formelfreier Überblicke über 
die historische Entwicklung:

http://arxiv.org/pdf/1208.2061v1.pdf

Und die Frage und Airys Experiment, ob sich was ändert, wenn man das 
Teleskop mit Wasser füllt (weil sich dann die Lichtgeschwindigkeit 
ändert):

http://arxiv.org/pdf/1401.5585v1.pdf

von Frank M. (ukw) (Moderator) Benutzerseite


Lesenswert?

Kurt Bindl schrieb:
> Als erstes schauen wir uns an wie so ein Polfilter aufgebaut ist und
> funktioniert.
>
> Es sind lauter Längsstreifen von Material das mit Licht in Resonanz
> gehen kann und das zugleich eine bestimmte Anordnung seiner einzelnen
> Resonanzkörper aufweist.
> Das ist nämlich Voraussetzung fürs Polfilter.

Du schwafelst schon wieder. Man kann durch Drehen[1] des 
Polarisationsfilters um die optische Achse Reflexionen von Glasscheiben 
und Wasseroberflächen wegfiltern.

Durch einfaches Drehen! Eine Longitudinalwelle ist aber um ihre optische 
Achse absolut symmetrisch! Das würde niemals klappen!

[1] Dabei durchläuft man beim Drehen um 360° jeweils zwei Maxima und 
zwei Minima des Filtereffektes.

: Bearbeitet durch Moderator
von Frank M. (ukw) (Moderator) Benutzerseite


Lesenswert?

A. K. schrieb:
> Kleine Anekdote zur Psychologie der Akzeptanz der Welt der RT. Lorentz,
> wahrlich kein unbeschriebenes Blatt in der Frage, hat die RT (und
> Einstein) sehr wohl aktiv unterstützt, ist aber trotzdem zeitlebens nie
> vom Äther weggekommen.

Ich verstehe nicht ganz, warum Kurt diesen von Dir[1] hier eingeworfenen 
Strohhalm mit Hinweis auf den Lorentzschen Äther nicht aufgegriffen hat. 
Damit hätte sich Kurt noch einigermaßen aus der Affäre ziehen können. 
Und Ähnlichkeiten zwischen dem Lorentz-Äther und dem Bindl-Träger sind 
ja durchaus vorhanden.

Aber Kurt ist ja gegen alles. Lorentz konnte mit seinem Äther durchaus 
relativistische Effekte erklären. Kurt kann das jedoch nicht.... 
jedenfalls nicht ohne abenteuerlichste Klimmzüge.

[1] 
Beitrag "Re: Vergleichen von Grössen"

von (prx) A. K. (prx)


Lesenswert?

Frank M. schrieb:
> Ich verstehe nicht ganz, warum Kurt diesen von Dir[1] hier eingeworfenen
> Strohhalm mit Hinweis auf den Lorentzschen Äther nicht aufgegriffen hat.

Er lehnt den Begriff des Inertialsystems allein deshalb ab und ersetzt 
ihn durch sein Bezugssystem, weil er IS nur aus dem Kontext der RT kennt 
und folglich implizit damit verbindet (siehe oben). Was meinst du wohl, 
wie es dann dem armen Herrn Lorentz ergeht, dem er sicherlich ebenfalls 
in der SRT zum ersten Mal begegnet ist.

Begrifflich gibts in der Lorentzschen Äthertheorie für ihn jede Menge 
Kröten aus der SRT zu schlucken, wie Lorentzkontraktion, -transformation 
und Zeitdilatation. Stark pointiert ist Einsteins SRT doch die 
Lorentzsche Theorie ohne Äther. Das kann nicht gutgehen. ;-)

Der ganze Zirkus der physikalischen Theorien des 19. Jahrhunderts bis 
SRT fusst massgeblich auf Maxwell, sowie der durch M&M beobachten 
Probleme mit der Addition von Geschwindigkeiten - ein damals durchaus 
unbefriedigendes Ergebnis. Daher zieht er es vor, M&M zu ignorieren bzw. 
für Unfug zu erklären, und Maxwell seine Transversalwellen zu nehmen.

> Aber Kurt ist ja gegen alles. Lorentz konnte mit seinem Äther durchaus
> relativistische Effekte erklären. Kurt kann das jedoch nicht....

Er will diese Effekte für Null und Nichtig erklären. Das kann er mit 
Lorentz nicht.

: Bearbeitet durch User
von Kurt B. (kurt-b)


Lesenswert?

Frank M. schrieb:
> A. K. schrieb:
>> Kleine Anekdote zur Psychologie der Akzeptanz der Welt der RT. Lorentz,
>> wahrlich kein unbeschriebenes Blatt in der Frage, hat die RT (und
>> Einstein) sehr wohl aktiv unterstützt, ist aber trotzdem zeitlebens nie
>> vom Äther weggekommen.
>
> Ich verstehe nicht ganz, warum Kurt diesen von Dir[1] hier eingeworfenen
> Strohhalm mit Hinweis auf den Lorentzschen Äther nicht aufgegriffen hat.
>
> Aber Kurt ist ja gegen alles. Lorentz konnte mit seinem Äther durchaus
> relativistische Effekte erklären. Kurt kann das jedoch nicht....
> jedenfalls nicht ohne abenteuerlichste Klimmzüge.
>
> [1]
> Beitrag "Re: Vergleichen von Grössen"

Ach was, warum soll ich nach Strohhalmen suchen, ich will nicht etwas 
das ich als ungenügend anschaue dazu verwenden etwas anderes das 
ebenfalls ungenügend ist, dazu verwenden mir scheinbare Probleme vom 
Laib zu halten.
Erst muss ich verstehen was bei der Aberration überhaupt alles 
angenommen wird und dann ist Zeit das in mein Überlegungen 
einzubeziehen.

Bisher (was sich aus dem was bei Wiki steht) scheint es so zu sein dass 
das Sonnensystem als ruhend zum Fixstern angenommen wird und die 
Bewegung der Erde dann betrachtet, nun neuerdings die Gravitationswerte 
mit eingerechnet werden.
Muss erst ein Bild zimmern damit ihr überhaut verstehen könnt was mir da 
so alles vorschwebt.

Zum Polfilter:
----------
Du schwafelst schon wieder. Man kann durch Drehen[1] des
Polarisationsfilters um die optische Achse Reflexionen von Glasscheiben
und Wasseroberflächen wegfiltern.

Durch einfaches Drehen! Eine Longitudinalwelle ist aber um ihre optische
Achse absolut symmetrisch! Das würde niemals klappen!
-------------

Selbstverständlich ist eine longitudinale Schwingung rundrum 
symmetrisch, damit kriegt man keine Polarisation hin.
Mit zwei symmetrischen Schwingungen, welche an unterschiedlichen Orten 
erzeugt und gegenphasig sind aber durchaus.

Die beiden Signale wirken bei ausgerichteten Antennen (Dipol) an 
unterschiedlichen Orten am Dipol mit gegenphasigem Signal.
Das regt die Resonanzschwingung des Dipols an.

Beim Polfilter sind die Dipole ausgerichtet (in Längsrichtung 
angeordnet)
und reagieren deswegen nur auf Licht das die richtige Polarisation hat.
Sie schwächen dann das Signal ab denn es sind ja viele Lagen 
hintereinander angeordnet (Dicke des Filters).

Gehen die Resonanzkörper/Dipolantennen nicht in Resonanz geht's einfach 
durch.


 Kurt

von (prx) A. K. (prx)


Lesenswert?

PS: In diversen Texten findet man M&M als Bestätigung der SRT. Das ist 
natürlich ein Zirkelschluss, denn die SRT haben wir massgeblich dem 
Schlamassel zu verdanken, in dem man dank M&M steckte. M&M: 1887, 
Lorentz: ab 1892, SRT: 1905. Bestätigungen gibts nachher, nicht vorher.

: Bearbeitet durch User
von Frank M. (ukw) (Moderator) Benutzerseite


Lesenswert?

Kurt Bindl schrieb:
> Beim Polfilter sind die Dipole ausgerichtet (in Längsrichtung
> angeordnet)
> und reagieren deswegen nur auf Licht das die richtige Polarisation hat.
> Sie schwächen dann das Signal ab denn es sind ja viele Lagen
> hintereinander angeordnet (Dicke des Filters).

Durch Drehen um die optische Achse??? Du hast den Knall noch nicht 
gehört.

Schau Dir doch mal das animierte Gif auf der Wikipedia-Seite an! Wenn Du 
das Filter um 90° drehst, wirkt es rabenschwarz, weil es das 
polarisierte Licht komplett filtert.

Du scheinst so ein Ding noch nie in der Hand gehabt zu haben. Anders 
kann ich mir nicht erklären, wie Du auf so einen Stuss kommst.

von (prx) A. K. (prx)


Lesenswert?

PPS, @Kurt: Man kann nicht wirklich behaupten, dass die Physiker über 
M&M glücklich waren. Ganz im Gegenteil, das hat ihnen die bestehende 
Physik zertrümmert. Infolgedessen wurde zweifellos ausgiebig nach Haaren 
in der Suppe gesucht.

von Kurt B. (kurt-b)


Lesenswert?

A. K. schrieb:
> PPS, @Kurt: Man kann nicht wirklich behaupten, dass die Physiker über
> M&M glücklich waren. Ganz im Gegenteil, das hat ihnen die bestehende
> Physik zertrümmert. Infolgedessen wurde zweifellos ausgiebig nach Haaren
> in der Suppe gesucht.

Verstehst du nun wieso ich nach Haaren beim MMI suche, denn es bestätigt 
genau das was ich behaupte, nämlich dass die Erde den Bezug für auf ihr 
und in ihrer Nähe laufenden Lichtes bereitstellt.
Und da will ich halt einigermassen sicher sein dass das auch durchs MMI 
richtig festgestellt, also ohne Fehler durch die Messeinrichtung selber, 
gemessen wurde.
Es wurden ja keine Nullwerte gemessen, sondern kleine Werte, diese 
kleinen Werte sind dann das was die Erde nicht schafft also das was noch 
übrig bleibt zu den 100%

Es wären halt Messergebnisse sinnvoll die in 10, 100, 1000, ... Km 
Abstand zur Erdoberfläche gemacht wurden um da zu sehen wieweit die 
Bezugsbildung reicht.

 Kurt

von Kurt B. (kurt-b)


Lesenswert?

Frank M. schrieb:
> Kurt Bindl schrieb:
>> Beim Polfilter sind die Dipole ausgerichtet (in Längsrichtung
>> angeordnet)
>> und reagieren deswegen nur auf Licht das die richtige Polarisation hat.
>> Sie schwächen dann das Signal ab denn es sind ja viele Lagen
>> hintereinander angeordnet (Dicke des Filters).
>
> Durch Drehen um die optische Achse??? Du hast den Knall noch nicht
> gehört.
>
> Schau Dir doch mal das animierte Gif auf der Wikipedia-Seite an! Wenn Du
> das Filter um 90° drehst, wirkt es rabenschwarz, weil es das
> polarisierte Licht komplett filtert.
>
> Du scheinst so ein Ding noch nie in der Hand gehabt zu haben. Anders
> kann ich mir nicht erklären, wie Du auf so einen Stuss kommst.

Wir müssen von vorne anfangen, anders geht's nicht, erst muss klar sein 
was am Dipol geschieht dann kanns zum Polfilter für Licht gehen.
(ein eigener Faden wäre hierzu wohl sinnvoll)

 Kurt

von Frank M. (ukw) (Moderator) Benutzerseite


Lesenswert?

Kurt Bindl schrieb:
> Verstehst du nun wieso ich nach Haaren beim MMI suche, [...]

A.K. wollte Dir damit sagen, dass dies schon wesentlich kompetentere 
Leute als Du das (erfolglos) versucht haben.

Erklär mir lieber, warum die Drehung(!) des Polfilters um die optische 
Achse eine longitudinale Welle wegfiltern soll. Deine Erklärungsversuch 
ist bisher kläglich gescheitert.

von (prx) A. K. (prx)


Lesenswert?

Kurt Bindl schrieb:
> Verstehst du nun wieso ich nach Haaren beim MMI suche

Sag ich doch. Ich verstehe, weshalb du danach suchst. Aber es ist die 
Suche nach dem Weihnachtsmann und dem Osterhasen.

von Kurt B. (kurt-b)


Lesenswert?

A. K. schrieb:
> Kurt Bindl schrieb:
>> Verstehst du nun wieso ich nach Haaren beim MMI suche
>
> Sag ich doch. Ich verstehe, weshalb du danach suchst. Aber es ist die
> Suche nach dem Weihnachtsmann und dem Osterhasen.

Und nach den Auswirkungen die Materie an Licht ausübt wenn dieses durch 
es hindurchgeht und mit ihm wechselwirkt.
Wecheselwirkungen sind zweifelsfrei vorhanden denn sonst gäbs keinen 
Spiegel, kein Filter, keinen Strahlteiler, keine Linse, keinen 
-Doppelspalt- ....
Also ist da WW da, die Frage ist wieweit sich diese aufs Messergebnis 
beim MMI auswirkt.


 Kurt

von Kurt B. (kurt-b)


Lesenswert?

Frank M. schrieb:
>
> Erklär mir lieber, warum die Drehung(!) des Polfilters um die optische
> Achse eine longitudinale Welle wegfiltern soll. Deine Erklärungsversuch
> ist bisher kläglich gescheitert.

Du kennst meine Erklärungen nicht, sie beginnen mit dem Dipol, mit dem 
was da vorgeht und wieso sich darauf dann Polarisation zeigt.


 Kurt

von Frank M. (ukw) (Moderator) Benutzerseite


Lesenswert?

Kurt Bindl schrieb:
> Du kennst meine Erklärungen nicht, sie beginnen mit dem Dipol, mit dem
> was da vorgeht und wieso sich darauf dann Polarisation zeigt.

Du redest von einem Dipol entlang der optischen Achse, ich drehe aber 
das Polfilter um(!) die optische Achse, um den Effekt zu mini- bzw. 
maximieren.

Das passt nicht!

Einen Polfilter bei longitudinalem Licht müsste ich vor und zurück 
bewegen. Dann würde auch Dein Argument mit der "Dicke" des Polfilters 
passen.

Aber das für Dich traurige: Ich muss ihn drehen! Also steht Dein Dipol 
senkrecht zur optischen Achse. Daraus folgt: Transversalwelle.

So einfach ist das.

: Bearbeitet durch Moderator
von Kurt B. (kurt-b)


Lesenswert?

Frank M. schrieb:
> Kurt Bindl schrieb:
>> Du kennst meine Erklärungen nicht, sie beginnen mit dem Dipol, mit dem
>> was da vorgeht und wieso sich darauf dann Polarisation zeigt.
>
> Du redest von einem Dipol entlang der optischen Achse, ich drehe aber
> das Polfilter um(!) die optische Achse, um den Effekt zu mini- bzw.
> maximieren.
>
> Das passt nicht!

Wir müssen von vorne anfangen (beim Dipol), anders geht's nicht.

 Kurt

(wenn du das Polfilter drehst drehst du die gestockten Dipole)
(die Dipole sind gestockt weil das Filter aus lauter "Fäden" besteht, 
erzeugt durch Ausrichtung beim Strecken)

.
.

von Kurt B. (kurt-b)


Lesenswert?

Frank M. schrieb:
> Kurt Bindl schrieb:
>> Du kennst meine Erklärungen nicht, sie beginnen mit dem Dipol, mit dem
>> was da vorgeht und wieso sich darauf dann Polarisation zeigt.
>
> Du redest von einem Dipol entlang der optischen Achse, ich drehe aber
> das Polfilter um(!) die optische Achse, um den Effekt zu mini- bzw.
> maximieren.
>
> Das passt nicht!
>
> Einen Polfilter bei longitudinalem Licht müsste ich vor und zurück
> bewegen. Dann würde auch Dein Argument mit der "Dicke" des Polfilters
> passen.
>
> Aber das für Dich traurige: Ich muss ihn drehen! Also steht Dein Dipol
> senkrecht zur optischen Achse. Daraus folgt: Transversalwelle.
>
> So einfach ist das.

Nein, ist es nicht denn du hast die Dipole falsch gelegt.
Wenn du das Filter drehst drehst du die Dipole in eine andere Ebene.

Dann sind sie als Dipole wirksam oder auch nicht.
Sind sie wirksam ist hintenraus dunkel, sind sie unwirksam hell.
Selbstverständlich gelten auch alle Zwischenwerte.

Wenn die Dipole aktiv sind wird das von ihnen abgefangene Signal 
letztendlich in Wärme verwandelt, und zwar deswegen weil die 
Nachbardipole (dicke des Filters) da auch angeregt werden.
Es ist wie eine Wand aus vielen Dipolen die alle in die selbe Richtung 
zeigen, gestockt sind und halt alles was da ist rausholen (oder eben 
auch nicht).


 Kurt

(ohne dass die einzelnen Resonanzkörper für diese Lichtfrequenzen aktiv 
werden ist hinten hell.
Umgedreht: werden die Resonanzkörper des Filters aktiv wird's hinterhalb 
dunkel und das Filter warm)

(Damit das gesittet abläuft müssen die einzelnen Resonanzkörper alle in 
die selbe Richtung ausgerichtet sein, das wird durch die "Fäden" des 
Filters bei seiner Herstellung bewerkstelligt)

.
.

von Frank M. (ukw) (Moderator) Benutzerseite


Lesenswert?

Kurt Bindl schrieb:
> Wir müssen von vorne anfangen (beim Dipol), anders geht's nicht.

Ja, Dein Dipol ist längs der optischen Achse ausgerichtet. Habe ich 
komplett verstanden! :-)

> (wenn du das Polfilter drehst drehst du die gestockten Dipole)

Was sind denn bitte "gestockte" Dipole???

> (die Dipole sind gestockt weil das Filter aus lauter "Fäden" besteht,

Was für Fäden? Wo sind da "Fäden" im Polfilter?

> erzeugt durch Ausrichtung beim Strecken)

Quatsch mit Soße. Was soll sich da "Strecken"??? Hör auf zu labern.

Nochmal:

Einen Polfilter bei longitudinalem Licht müsste ich vor und zurück
bewegen, um den Effekt ein- und auszuschalten. Dann würde auch Dein 
Argument mit der "Dicke" des Polfilters passen. Meinetwegen auch Deine 
"Fäden" - die ich nicht verstehe. Da sind keine Fäden.

Aber die Realität sieht anders aus:

Ich muss den Filter drehen! Also steht Dein Dipol senkrecht zur 
optischen Achse. Daraus folgt: Transversalwelle.

Du kannst Dir das auch anders vorstellen:

Ein Kind hat ein Seil an eine Wand angebunden. Es erzeugt mit 
senkrechten Ausschlägen des Seilendes eine stehende Welle. Soweit klar?

Jetzt nimmt ein zweites Kind zwei Besenstiele und stellt jeweils 
senkrecht einen Besenstiel links vom Seil ab, den anderen rechts.

Was passiert? Richtig: Gar nichts, denn die Stiele behindern ja nicht 
das Seil, weil sie ebenso senkrecht stehen wie die Welle. Jetzt dreht 
das Kind die beiden Besenstiele langsam in die Waagerechte.... Die 
Schwingung des Seiles wird am Ende auf 0 gebremst.

Und genau DAS passiert beim Drehen des Polfilters.

Der Dipol steht folglich senkrecht zur optischen Achse, also 
Transversalwelle.

: Bearbeitet durch Moderator
von Robert L. (lrlr)


Lesenswert?

@Kurt:
>Verstehst du nun wieso ich nach Haaren beim MMI suche, denn es bestätigt
>genau das was ich behaupte, nämlich dass die Erde den Bezug für auf ihr
>und in ihrer Nähe laufenden Lichtes bereitstellt.

d.h. das wäre auch der "Bezug" für Licht IM (bzw. Um ein) Flugzeug, oder 
hat das einen eigen "Bezug" ?

von (prx) A. K. (prx)


Lesenswert?

Zu den Polfiltern: Ist wie mit Schaltbildern oder Platinen, um im 
offtopic mal offtopic zu sein: Auch die lassen sich in Bildern besser 
beschreiben als in Worten.

: Bearbeitet durch User
von Kurt B. (kurt-b)


Lesenswert?

Robert L. schrieb:
> @Kurt:
>>Verstehst du nun wieso ich nach Haaren beim MMI suche, denn es bestätigt
>>genau das was ich behaupte, nämlich dass die Erde den Bezug für auf ihr
>>und in ihrer Nähe laufenden Lichtes bereitstellt.
>
> d.h. das wäre auch der "Bezug" für Licht IM (bzw. Um ein) Flugzeug, oder
> hat das einen eigen "Bezug" ?

Es hat seinen eigenen Bezug, der ist halt so gross wie es die paar BT 
des Fliegers eben hergeben/schaffen.

Kannst vergessen weil zu klein um gemessen zu werden.

Fizeau hats ja gezeigt.
Das was das Lichtlaufen da bestimmt ist die Mischung aus:
- Bezug den die Masse der Erde erbringt
- Bezug den das bewegte Wasser hervorbringt.

Das Wasser ist zwar näher am Licht, jedoch verschwindend klein in seinem 
Verhältnis zur Masse der Erde.

Darum bewegt sich das Licht in den Rohren nicht in Bezug zu dem es 
umgebenden Wassers, sondern dem Bezug den die Erde und das Wasser 
gemeinsam hervorbringen.


 Kurt

von Frank M. (ukw) (Moderator) Benutzerseite


Lesenswert?

A. K. schrieb:
> Zu den Polfiltern: Ist wie mit Schaltbildern oder Platinen, um im
> offtopic mal offtopic zu sein: Auch die lassen sich in Bildern besser
> beschreiben als in Worten.

Ich habe ihm einen Link auf den Wikipedia-Artikel und einen Hinweis auf 
das animierte GIF oben rechts gegeben, wo man die Drehung und den Effekt 
schön sehen kann. Was soll ich noch tun? Einem Betonklotz gut zureden?

von Kurt B. (kurt-b)


Lesenswert?

A. K. schrieb:
> Zu den Polfiltern: Ist wie mit Schaltbildern oder Platinen, um im
> offtopic mal offtopic zu sein: Auch die lassen sich in Bildern besser
> beschreiben als in Worten.

Mir wird nichts anderes übrig bleiben als zu malen.

Zum "Gestockte Dipole" und "Fäden".

Um eine flache Abstrahlung oder einen Empfang mit Gewinn zu erreichen 
werden mehrere Dipole senkrecht übereinander angeordnet.
Das heisst dann -gestockte- Antennen/Dipole usw.


Die Fäden des Filters sind auch gestockte Dipole denn da werden 
langkettige Moleküle, welche mikroskopische Antennen sind (die 
Resonanzkörper fürs Licht) während der Herstellung des Filters 
mechanisch oder auch elektrisch so ausgerichtet das sie alle in der 
selben Richtung liegen.
Das sind dann die -Fäden-, oder die gestockten Dipole von denen ich 
rede.

Diese gehen gemeinsam dem Licht zuleibe oder garnicht, eben je nach 
Polarisationsebene.

 Kurt

von (prx) A. K. (prx)


Lesenswert?

Kurt Bindl schrieb:
> Mir wird nichts anderes übrig bleiben als zu malen.

So war's gemeint.

> Zum "Gestockte Dipole" und "Fäden".

Aber bitte nicht in Worten. ;-)

von Frank M. (ukw) (Moderator) Benutzerseite


Lesenswert?

Kurt Bindl schrieb:
> A. K. schrieb:
>> Zu den Polfiltern: Ist wie mit Schaltbildern oder Platinen, um im
>> offtopic mal offtopic zu sein: Auch die lassen sich in Bildern besser
>> beschreiben als in Worten.
>
> Mir wird nichts anderes übrig bleiben als zu malen.

Ja, male mal.

Und frage Dich dabei, warum Autoantennen senkrecht stehen oder warum 
Autohersteller dermaßen bekloppt sein müssen.

von J.-u. G. (juwe)


Lesenswert?

Frank M. schrieb:
> Ja, male mal.

Aber Kurt hat doch seinerzeit im Selektivgitterthread so schön Skizzen 
zu dieser Thematik gezeigt:
Beitrag "Re: Selektivgitter"

Und zwar unter Verwendung zahlreicher "kleiner als"- und "größer 
als"-Zeichen. Und damit passend zum Ausgangsposting des aktuellen 
Threads.

von Kurt B. (kurt-b)


Lesenswert?

Frank M. schrieb:
> A. K. schrieb:
>> Zu den Polfiltern: Ist wie mit Schaltbildern oder Platinen, um im
>> offtopic mal offtopic zu sein: Auch die lassen sich in Bildern besser
>> beschreiben als in Worten.
>
> Ich habe ihm einen Link auf den Wikipedia-Artikel und einen Hinweis auf
> das animierte GIF oben rechts gegeben, wo man die Drehung und den Effekt
> schön sehen kann. Was soll ich noch tun? Einem Betonklotz gut zureden?

Mensch Mann, ich habe mir zwei eigenständig drehbare Filter auf einen 
Plastikklotz geklebt und kann sehen was hinten passiert/noch rauskommt.

Ich kann sehen was durchgeht, was gespiegelt wird und auch auf welche 
Frequenzen sie wie wirken.
Es fehlt nur noch der Temperatursensor der aufzeigt wieviel Wärme da 
wann entsteht.


 Kurt

von Frank M. (ukw) (Moderator) Benutzerseite


Lesenswert?

J.-u. G. schrieb:
> Aber Kurt hat doch seinerzeit im Selektivgitterthread so schön Skizzen
> zu dieser Thematik gezeigt:
> Beitrag "Re: Selektivgitter"

Das ist nichts als Geschmiere. Ein dreijähriges Kind kann das besser.

von Frank M. (ukw) (Moderator) Benutzerseite


Lesenswert?

Kurt Bindl schrieb:
> Mensch Mann, ich habe mir zwei eigenständig drehbare Filter auf einen
> Plastikklotz geklebt und kann sehen was hinten passiert/noch rauskommt.

Na also! Mach mal ein Foto!

Wenn Du mich nun mittels dieses Aufbaus überzeugen kannst, dass 
Lichtwellen longitudinal sind, dann werde ich

  1. Meine Autoantenne direkt waagerecht aufs Auto schrauben

  2. Nur noch in eine Richtung fahren, damit der Sender auch gut
     "reinkommt".

von Kurt B. (kurt-b)


Lesenswert?

Frank M. schrieb:
> Kurt Bindl schrieb:
>> A. K. schrieb:
>>> Zu den Polfiltern: Ist wie mit Schaltbildern oder Platinen, um im
>>> offtopic mal offtopic zu sein: Auch die lassen sich in Bildern besser
>>> beschreiben als in Worten.
>>
>> Mir wird nichts anderes übrig bleiben als zu malen.
>
> Ja, male mal.
>
> Und frage Dich dabei, warum Autoantennen senkrecht stehen oder warum
> Autohersteller dermaßen bekloppt sein müssen.

Sind sie das wirklich?
Oder versuchen sie die Radioantenne für UKW möglichst flach liegend 
hinzubekommen oder sie an eine Kante legen, denn da wird die 
Polarisation nämlich gedreht und somit das Signal erst empfangbar.

Falls du von der GPS-Antenne redest, die ist als Flachstrahler mit 
möglichst wenig Polarisationsempfindlichkeit ausgelegt damits die SATs 
kriegt.

Bei GSM steht diese senkrecht denn die Stationsantennen 
strahlen/empfangen dort auch vertikal.

Wenn du die Naiverklärung mit den Gitterstäben, wo die Transversalwelle 
sich mal ungehindert durchdrängt und mal an den Stäben sich spisst, als 
korrekt anschaust, dann frag ich dich ab welchem Abstand das passiert 
und wie hoch dann die Amplitude der Transversalwelle in cm sein darf 
damit sie (auch bei falscher Ausrichtung) noch durchkommt.


 Kurt

von Frank M. (ukw) (Moderator) Benutzerseite


Lesenswert?

Kurt Bindl schrieb:
> Oder versuchen sie die Radioantenne für UKW möglichst flach liegend
> hinzubekommen oder sie an eine Kante legen, denn da wird die
> Polarisation nämlich gedreht und somit das Signal erst empfangbar.

Ja, ich meine UKW-Autoradioantenne. Ich meine den Stab, der möglichst 
senkrecht steht, von einigen Autoherstellern wegen des Gegenwindes etwas 
geneigt wird.

Nur, wenn Du ihn senkrecht stellst, bist Du davon unabhängig in welcher 
Richtung (innerhalb der Erdoberflächen-Ebene) der Sender steht. Wenn Du 
den Stab legst, hast Du nur (einigermaßen) guten Empfang, wenn die 
UKW-Wellen senkrecht auf den Stab treffen. Zeige ich mit dem 
Antennenstab auf den Sender, ist der Empfang minimal bis gar nicht da.

Also: Transversalwelle.

(Rest wie GPS usw. wegen Irrelevanz gelöscht).

Und jetzt komm nicht mit irgendeinem Scheiß, ja Du hättest schon zwei 
Autos... und Du hättest Sie auf einem Plastikklotz schon mal drehbar 
zueinander angeordnet und hast dabei festgestellt, dass UKW-Sender Hitze 
erzeugen.

: Bearbeitet durch Moderator
von Robert L. (lrlr)


Lesenswert?

Kurt Bindl schrieb:
>> d.h. das wäre auch der "Bezug" für Licht IM (bzw. Um ein) Flugzeug, oder
>> hat das einen eigen "Bezug" ?
>
> Es hat seinen eigenen Bezug, der ist halt so gross wie es die paar BT
> des Fliegers eben hergeben/schaffen.
>
> Kannst vergessen weil zu klein um gemessen zu werden.

falls ich das richtig verstanden habe, glaubst du also dass licht 
(taschenlampe) im flugzeug sich nach vorne langsamer ausbreitet als nach 
hinten..

von Kurt B. (kurt-b)


Lesenswert?

Robert L. schrieb:
> Kurt Bindl schrieb:
>>> d.h. das wäre auch der "Bezug" für Licht IM (bzw. Um ein) Flugzeug, oder
>>> hat das einen eigen "Bezug" ?
>>
>> Es hat seinen eigenen Bezug, der ist halt so gross wie es die paar BT
>> des Fliegers eben hergeben/schaffen.
>>
>> Kannst vergessen weil zu klein um gemessen zu werden.
>
> falls ich das richtig verstanden habe, glaubst du also dass licht
> (taschenlampe) im flugzeug sich nach vorne langsamer ausbreitet als nach
> hinten..

Diese Frage lässt sich nicht so ohne weiteres eindeutig/klar 
beantworten, denn es fehlt der Bezug gegen den diese Frage aussagen 
soll.

Wenn du das Flugzeug als Bezug für die Signalgeschwindigkeitsangabe 
nimmst, dieses gegen den LlBezug bewegt ist, dann ja.

(es ist nicht nur das Licht der Taschenlampe, sonders jedes das da 
läuft, egal woher es kommt es muss nur hinten rein können)


 Kurt

von Frank M. (ukw) (Moderator) Benutzerseite


Lesenswert?

Kurt Bindl schrieb:
> Wenn du das Flugzeug als Bezug für die Signalgeschwindigkeitsangabe
> nimmst, dieses gegen den LlBezug bewegt ist, dann ja.

Wo sitzt der Beobachter? Im Flugzeug oder auf der Erde?

von Robert L. (lrlr)


Lesenswert?

Kurt Bindl schrieb:
> Diese Frage lässt sich nicht so ohne weiteres eindeutig/klar
> beantworten, denn es fehlt der Bezug gegen den diese Frage aussagen
> soll.
>
> Wenn du das Flugzeug als Bezug für die Signalgeschwindigkeitsangabe
> nimmst, dieses gegen den LlBezug bewegt ist, dann ja.
>

kleine Klarstellung, das Flugzeug fliegt (in der Luft) auf der Erde, am 
equatort, gegen die erdrotation , ein paar 100m über der Erdoberfläche
alle Messgeräte, Beobachter, Taschenlampen sind IM flugzeug..(und 
bewegen sich relativ zum Flugzeug nicht..)

kannst das JETZT mit ja/nein beantworten?

: Bearbeitet durch User
von Kurt B. (kurt-b)


Lesenswert?

Robert L. schrieb:
> Kurt Bindl schrieb:
>> Diese Frage lässt sich nicht so ohne weiteres eindeutig/klar
>> beantworten, denn es fehlt der Bezug gegen den diese Frage aussagen
>> soll.
>>
>> Wenn du das Flugzeug als Bezug für die Signalgeschwindigkeitsangabe
>> nimmst, dieses gegen den LlBezug bewegt ist, dann ja.
>>
>
> kleine Klarstellung, das Flugzeug fliegt (in der Luft) auf der Erde, am
> equatort, gegen die erdrotation , ein paar 100m über der Erdoberfläche
> alle Messgeräte, Beobachter, Taschenlampen sind IM flugzeug..(und
> bewegen sich relativ zum Flugzeug nicht..)
>
> kannst das JETZT mit ja/nein beantworten?


Die Erdrotation lassen wir mal weg denn sonst kommt Sagnac auch noch an 
Board.

Dann ist die Aussage eindeutig erstellbar.
Von hinten nach vorne ist das Licht länger unterwegs als von vorne nach 
hinten.


 Kurt

(hat jemand ein transportables MMI zur Hand, das wäre jetzt der richtige 
Zeitpunkt um das an Board zu nehmen)

.
.

von Kurt B. (kurt-b)


Lesenswert?

Frank M. schrieb:
> Kurt Bindl schrieb:
>> Oder versuchen sie die Radioantenne für UKW möglichst flach liegend
>> hinzubekommen oder sie an eine Kante legen, denn da wird die
>> Polarisation nämlich gedreht und somit das Signal erst empfangbar.
>
> Ja, ich meine UKW-Autoradioantenne. Ich meine den Stab, der möglichst
> senkrecht steht, von einigen Autoherstellern wegen des Gegenwindes etwas
> geneigt wird.
>
> Nur, wenn Du ihn senkrecht stellst, bist Du davon unabhängig in welcher
> Richtung (innerhalb der Erdoberflächen-Ebene) der Sender steht. Wenn Du
> den Stab legst, hast Du nur (einigermaßen) guten Empfang, wenn die
> UKW-Wellen senkrecht auf den Stab treffen. Zeige ich mit dem
> Antennenstab auf den Sender, ist der Empfang minimal bis gar nicht da.
>
> Also: Transversalwelle.

Robert, gehe ich richtig in der Annahme dass du noch nichts mit Funk 
gemacht hast, denn dann müsste dir klar sein warum die UKW Antenne 
möglichst waagrecht sein sollte, denn die Polarisation der UKW-Sender 
ist Horizontal.
Auch würdest du dann die GPS und die GSM-Antenne nicht wegstreichen den 
GPS ist zirkular und GSM vertikal polarisiert.

Machst du die UKW-Antenne möglichst in der Mitte des Autodaches hin 
(ergibt eine gute GP) dann bekommst du zwar Sender rein, diese sind aber 
um 20 dB schlechter da als wie wenn die Antenne an einem 
Polarisationsdrehenden Holm entlang geführt wird.
Das hat zwar immer noch eine gewisse Richtwirkung, aber wegen der 
Windlast wird wohl kein Autohersteller die Antenne schräg legen.


 Kurt


(wenn du so sehr von der Transversalwelle überzeugt bist dann kannst du 
mir ja sicherlich Angaben zu deren Amplituden bei gewissen 
Sendeleistungen in einer bestimmten Entfernung zum Sendedipol machen)
(der Amplitude der elektrischen- und der magnetischen Welle)

.
.

von Kurt B. (kurt-b)


Lesenswert?

Frank M. schrieb:
> Kurt Bindl schrieb:
>> Wenn du das Flugzeug als Bezug für die Signalgeschwindigkeitsangabe
>> nimmst, dieses gegen den LlBezug bewegt ist, dann ja.
>
> Wo sitzt der Beobachter? Im Flugzeug oder auf der Erde?

Darum hab ich ja nachgehakt um klare Verhältnisse zu haben/kennen.

 Kurt

von Robert L. (lrlr)


Lesenswert?

Kurt Bindl schrieb:
> Dann ist die Aussage eindeutig erstellbar.
> Von hinten nach vorne ist das Licht länger unterwegs als von vorne nach
> hinten.

das würde aber der RT widersprechen, und auch leicht messbar sein (wied 
du ja selber schreibst )..
wäre die RT so einfach zu widerlegen, hätts wohl schon jemand gemacht..

: Bearbeitet durch User
von Kurt B. (kurt-b)


Lesenswert?

Robert L. schrieb:
> Kurt Bindl schrieb:
>> Dann ist die Aussage eindeutig erstellbar.
>> Von hinten nach vorne ist das Licht länger unterwegs als von vorne nach
>> hinten.
>
> das würde aber der RT widersprechen, und auch leicht messbar sein (wied
> du ja selber schreibst )..
> wäre die RT so einfach zu widerlegen, hätts wohl schon jemand gemacht..

Hat aber noch niemand.

 Kurt

von Frank M. (ukw) (Moderator) Benutzerseite


Lesenswert?

Kurt Bindl schrieb:
> (hat jemand ein transportables MMI zur Hand, das wäre jetzt der richtige
> Zeitpunkt um das an Board zu nehmen)

Ich habe Dich schon vor Tagen gefragt, ob ein MMI auf dem Mond dieselben 
Ergebnisse bringt. Du hast die Frage zweimal geflissentlich ignoriert.

Was soll das, ein MMI ins Flugzeug zu stecken? Unsere Erde ist auch ein 
Flugzeug. Oder willst Du sagen, dass die Erde etwas ausgezeichnetes ist? 
Nicht Dein Ernst, oder?

von Frank M. (ukw) (Moderator) Benutzerseite


Lesenswert?

Kurt Bindl schrieb:
>> wäre die RT so einfach zu widerlegen, hätts wohl schon jemand gemacht..
>
> Hat aber noch niemand.

Dann machs.

von Kurt B. (kurt-b)


Lesenswert?

Frank M. schrieb:
> Kurt Bindl schrieb:
>> (hat jemand ein transportables MMI zur Hand, das wäre jetzt der richtige
>> Zeitpunkt um das an Board zu nehmen)
>
> Ich habe Dich schon vor Tagen gefragt, ob ein MMI auf dem Mond dieselben
> Ergebnisse bringt. Du hast die Frage zweimal geflissentlich ignoriert.
>

Es bringt im Prinzip Ähnliches, es wäre echt interessant diese Werte zu 
kennen.


> Was soll das, ein MMI ins Flugzeug zu stecken? Unsere Erde ist auch ein
> Flugzeug. Oder willst Du sagen, dass die Erde etwas ausgezeichnetes ist?
> Nicht Dein Ernst, oder?

Wenn du das so siehst dann haben wir die letzten Tage aneinander 
vorbeigeredet.

(ein MMI verwendet als Bezug fürs Lichtlaufen in ihm den Bz_l)


 Kurt

von Kurt B. (kurt-b)


Lesenswert?

Frank M. schrieb:
> Kurt Bindl schrieb:
>>> wäre die RT so einfach zu widerlegen, hätts wohl schon jemand gemacht..
>>
>> Hat aber noch niemand.
>
> Dann machs.

Gerne, wenn nur diese verflixte Realität nicht wäre.

 Kurt

von Frank M. (ukw) (Moderator) Benutzerseite


Lesenswert?

Kurt Bindl schrieb:
> Wenn du das so siehst dann haben wir die letzten Tage aneinander
> vorbeigeredet.

Natürlich haben wir das.

> (ein MMI verwendet als Bezug fürs Lichtlaufen in ihm den Bz_l)

Was ist ein Bz_l? Ich habe keine Lust, für Dich eine Legende Deiner 
allgemein absolut unüblichen Abkürzungen anzulegen. Sprich Deutsch oder 
verwende zumindest allgemein bekannte Begriffe.

Du gibst jeder Sache, die Du auf Deine Art und Weise erklären willst, 
einen neuen Namen. Und die soll dann ein anderer verstehen?

Wie wär's mit einem Bindlschen Lexikon:

BT=
Tr=
Bz=
Bz_l=
usw. usw.

Einfach nur ätzend. Solange Du Dir keine Mühe gibst, Dich verständlich 
auszudrücken, werde ich bestimmt keine Anstrengungen unternehmen, Dich 
nur annähernd zu verstehen.

von Kurt B. (kurt-b)


Lesenswert?

Frank M. schrieb:

>
> Wie wär's mit einem Bindlschen Lexikon:
>
> BT=
> Tr=
> Bz=
> Bz_l=
> usw. usw.
>
> Einfach nur ätzend. Solange Du Dir keine Mühe gibst, Dich verständlich
> auszudrücken, werde ich bestimmt keine Anstrengungen unternehmen, Dich
> nur annähernd zu verstehen.


 BT= Basisteilchen, Grundbaustein der Materie

 Tr= Träger, Abkürzung für das -Ding-, die Substanz (Trägersubstanz) die 
unser Universum ausmacht/bildet

 Bz= Bezug, Angaben zu Grössen oder vergleichenden Aussagen, solche sind 
immer auf einen Bezug zu beziehen, ansonsten sind sie wertlos und 
unbrauchbar


 Bz_l=   Bezug_lichtlaufen, das ist der Bezug den Licht verwendet wenn 
es sich ausbreitet


 Bz_m=   Bezug_materie (verhalten), Bezug für das Verhalten von Materie


 Kurt

von Kurt B. (kurt-b)


Lesenswert?

Frank M. schrieb:
> Kurt Bindl schrieb:
>> Wenn du das Flugzeug als Bezug für die Signalgeschwindigkeitsangabe
>> nimmst, dieses gegen den LlBezug bewegt ist, dann ja.
>
> Wo sitzt der Beobachter? Im Flugzeug oder auf der Erde?

Der Beobachter ist der der hier die Angabe zur Geschwindigkeit gibt.
In diesem Fall hier ist er in Flugzeug und misst die Laufzeit vom Licht 
dass mal so, mal so läuft.
Er benutzt also eine Strecke die er im Flugzeug festlegt und schaut wie 
lange es jeweils unterwegs ist.
Und dabei ergeben sich unterschiedliche Laufzeiten.
Ganz einfach deswegen weil das Lichtsignal unterschiedlich lange 
Laufstrecken in seinem Bezug durchlaufen muss.


 Kurt

von Kurt B. (kurt-b)


Lesenswert?

J.-u. G. schrieb:
> Frank M. schrieb:
>> Ja, male mal.
>
> Aber Kurt hat doch seinerzeit im Selektivgitterthread so schön Skizzen
> zu dieser Thematik gezeigt:
> Beitrag "Re: Selektivgitter"
>
> Und zwar unter Verwendung zahlreicher "kleiner als"- und "größer
> als"-Zeichen.

Ja richtig, das hatten wir doch schon mal, ist davon nichts hängen 
geblieben?

Um Polarisation zu bekommen sind zwei Signale notwendig, diese werden am 
Dipol erzeugt und breiten sich im Medium aus, treffen sie auf -passende- 
Dipole geht's wieder rückwärts, es baut sich eine resonante Schwingung 
auf.
Liegt der Empfangsdipol in der -falschen- Ebene kann sich keine Resonanz 
auf/an/mit ihm aufbauen.

So einfach ist Polarisation.

Beim Polfilter für Licht sind es halt viele Dipole die, als solche in 
der richtigen Ebene angeordnet, ebenfalls ihren Dienst leisten.

Da ist doch wirklich nichts Besondres dabei.

Die naiven Vorstellungen mit dem Gitter das die Amplitude der 
vermeintlichen Transversalschwingung einfach sperrt weil diese halt 
nicht durchschlüpfen kann sind einfach nur lächerlich.
(wie ists denn mit dem -magnetischem- Teil des Signals, dreht sich 
dieses um 90 Grad wenns am Filter/Gitterstäben ankommt damit es auch 
nicht anstösst?)


 Kurt

(das Verständnis dessen was am Dipol abläuft ist die Voraussetzung um zu 
verstehen wie Polarisation entsteht, was Polarisiert überhaupt bedeutet)

Diejenigen unter euch die Ahnung von HF haben werden mir sicherlich 
zustimmen dass die Beschreibung wie denn so eine Transversalwelle, 
bestehend aus einen -elektrischen- und einem, 90° versetztem 
-magnetischen- Signal aus einem LC-Schwingkreis entstehen soll, mehr als 
peinlich ist.
Bedenkt einfach mal die Proportionen für L und C.

Würde das so sein wie in Büchern beschrieben dann würde sich so mancher 
im Grab umdrehen denn da müsste instantane Informationsübermittlung am 
Dipol stattfinden.
Ist aber nicht.

 .
.

von Kurt B. (kurt-b)


Angehängte Dateien:

Lesenswert?

Aberration:

So ähnlich stelle ich mir die Lichtablenkung der Erde vor,
Bezug0 ist der den die Sonne bereitstellt, die anderen sind für den 
einen der Erde gezeichnet.

Kommt Licht an den Polen rein dann wird es nicht abgelenkt.
Die Ablenkung die das Signal auf ihrem Weg ins Sonnensystem erleidet (an 
grossen Massen vorbei), sind hier nicht eingezeichnet.


 Kurt

von (prx) A. K. (prx)


Lesenswert?

Gilt dieses "Mitschleppen" der Lichtleitung für jede Bewegung der Erde? 
Andernfalls käme die jährliche Aberration unter die Räder.

Ist das masseabhängig? Also die Erde schleppt mit ihrer 
Eigengeschwindigkeit mit, ein Auto aber nicht?

: Bearbeitet durch User
von Kurt B. (kurt-b)


Lesenswert?

A. K. schrieb:
> Gilt dieses "Mitschleppen" der Lichtleitung für jede Bewegung der Erde?
> Andernfalls käme die jährliche Aberration unter die Räder.
>

Die Masse der Erde beeinflusst ihr Umfeld, also ist der Bezug sozusagen 
an sie angebunden.
Bewegt sie sich um die Sonne geht der natürlich mit.


> Ist das masseabhängig? Also die Erde schleppt mit ihrer
> Eigengeschwindigkeit mit, ein Auto aber nicht?

Doch auch ein Auto macht das, ist bei Fizeau zu sehen, da macht es das 
Wasser das da gegen die Masse der Erde bewegt ist.
Nur sind die Verhältnisse halt sehr unterschiedlich.

 Kurt

----
Beim GPS wird bei der Systemzeitübermittlung die Erdrotation 
rausgerechnet.
Ich bin mir nicht sicher ob das richtig ist denn es würde bedeuten dass 
die Erde zwar den Bezug um sich herum bereitstellt, jedoch sich darin 
dann wiederum selber dreht.
Das ergibt dann sozusagen gekrümmte N/S-Linien für die einzelnen 
Zeitzonenbereiche der Erde.

Ist ein wenig seltsam, denn das MMI zeigt was anderes.
----

.
.

von Kurt B. (kurt-b)


Lesenswert?

Nochwas zum -Flugzeug-.

Das Flugzeug sei am Boden und werde ein Zug, dieser Zug ist 1 LS, also 
300 000 Km lang.

Er bewege sich aufnem Gleis auf der Erde mit 1/2 c, also mit 150 000 
Km/s von links nach rechts.

Sobald er mit seinem Ende am Punkt "S" (für Sender) vorbeikommt wird 
dort ein Lichtblitz erzeugt.
Dieser läuft nun im Zug nach vorne zum "E" (Empfänger im Zug), wenn er 
dort ankommt wird eine Markierung am Gleis gesetzt.


- wie weit ist die Markierung vom "S" Punkt weg?
- wie lange war das Signal unterwegs bis es beim "E" ankam?


.....S----------Zug----------E
=====S==============================================================Glei 
s
Bewegungsrichtung >>>


 Kurt

von Robert L. (lrlr)


Lesenswert?

ich rate mal:


von S nach E, im Zug, für betrachter/messung im Zug 1 Sekunde..
markierung bei 600000

(von extern betrachtet fährt er also 2 sekunden.)

von Kurt B. (kurt-b)


Lesenswert?

Robert L. schrieb:
> ich rate mal:
>
>
> von S nach E, im Zug, für betrachter/messung im Zug 1 Sekunde..
> markierung bei 600000
>
> (von extern betrachtet fährt er also 2 sekunden.)


Nun spendieren wir dem Zug noch einen sog. -Beobachter-, nämlich Uhren.
Diese wurden vorher so abgeglichen das sie mit den Uhren die stationär 
am/beim Gleis sind identisch takten, und zwar dann wenn der Zug fährt.
Alle Uhren erstellen also identische Zeiteinheiten (sec usw.).



.....S---------U1-Zug-U2---------E
=====S===========================================================Gleis

Bewegungsrichtung Zug  >>>
Bewegungsrichtung Signal >>

U1 und U2 sind 300 Km auseinander, ebenso Uhren die am Gleis stehen,
dass Signal, am Gleis beobachtet, braucht für diese Strecke 1 ms.

Frage: wie lange braucht es im Zug für die Strecke zwischen U1 und U2?


 Kurt

von Robert L. (lrlr)


Lesenswert?

> Gleis sind identisch takten, und zwar dann wenn der Zug fährt.

wie (technisch) soll das gehen?

von Robert L. (lrlr)


Lesenswert?

ok, ich beantworte es mal selber:
Da der zug c/2 fährt macht man alle 150000 eine Markierung
von extern betrachtet fährt der Zug jede Sekunden über eine Markierung.

Im Zug ist ein Gerät welches jedes mal wenn es über die Markierung fährt 
einen Impuls abgibt (das ist das was du als "Uhren im Zug" bezeichnest)
ist aber keine Uhr die Sekunden anzeigt.
Eine Sekunde ist einfach nicht so definiert..

Stellt man eine Atomuhr daneben (die dann tatsächlich Sekunden liefert), 
wird man feststellen,  dass man nicht jede Sekunde einen Impuls erhält.

>Frage: wie lange braucht es im Zug für die Strecke zwischen U1 und U2?

Gegenfrage: auf welcher Uhr oder "Uhr" oder -Uhr- abgelesen?

: Bearbeitet durch User
von Kurt B. (kurt-b)


Lesenswert?

Robert L. schrieb:
> ok, ich beantworte es mal selber:
> Da der zug c/2 fährt macht man alle 150000 eine Markierung
> von extern betrachtet fährt der Zug jede Sekunden über eine Markierung.
>

Ist eine Möglichkeit, mir fallen bestimmt noch weitere ein.

Ich denke da spontan an einen Regelkreis der die Zuguhr(en), deren 
Oszillator, so einregelt dass es an den Schwellen passt, denn diese 
können ja so gesetzt werden dass sie in ein Zeitraster passen und damit 
dann der Uhroszillator im Zug eingeregelt werden kann.
Dann tackten alle Uhren in gleichem zeitlichem Abstand.

> Eine Sekunde ist einfach nicht so definiert..

Wir verwenden einfach unsere eigene, ich schlage vor die vom 
GPS-Zeitsystem zu verwenden.

>
>>Frage: wie lange braucht es im Zug für die Strecke zwischen U1 und U2?
>
> Gegenfrage: auf welcher Uhr oder "Uhr" oder -Uhr- abgelesen?

Nachdem sie ja alle gleich gehen ists egal, es muss auch nicht direkt 
abgelesen werden, sondern es reicht wenn die Uhrenwerte der jeweiligen 
Ereignisse bekannt sind, diese werden einfach abgelegt oder über Funk 
oder sonstwie übermittelt und können dann in aller Ruhe, ohne 
irgendwelche Laufzeitüberlegungen anstellen zu müssen, ausgewertet 
werden.

Die Uhren müssen auch nicht synchronisiert sein, es reicht ja die 
Differenzen der relevanten Ereignisse auszuwerten.


 Kurt

von Robert L. (lrlr)


Lesenswert?

>Nachdem sie ja alle gleich gehen ists egal,

nein, hab ich dir doch erklärt
die atomuhr im zug, und das ding was du als "uhr" bezeichnest, gehen 
nicht gleich..

von Kurt B. (kurt-b)


Lesenswert?

Robert L. schrieb:
>>Nachdem sie ja alle gleich gehen ists egal,
>
> nein, hab ich dir doch erklärt
> die atomuhr im zug, und das ding was du als "uhr" bezeichnest, gehen
> nicht gleich..

Wir haben sie ja durch eine PLL gleichgehend gemacht, synchron brauchen 
sie ja nicht zu gehen.

Es reicht ja aus Differenzanzeigen auszuwerten, wie diese schadfrei zur 
Auswertung gelangen ist ja auch geklärt.


 Kurt

von Robert L. (lrlr)


Lesenswert?

ich weiß nicht was du willst/schreib doch mal verständliche Sätze!..


für den Betrachter am Bahnhof vergehen dann 100 Takte, und für den im 
Zug auch 100 Takte

für den Betrachter am Bahnhof dauert ein Takt eine Sekunde
für den Betrachter im Zug nicht

von Kurt B. (kurt-b)


Lesenswert?

Robert L. schrieb:
> ich weiß nicht was du willst/schreib doch mal verständliche Sätze!..
>

Ist was an meinen Sätzen unverständlich, verstehst du den Begriff PLL 
zur Gleichschaltung der Uhrenoszillatoren nicht oder den der GPS-Zeit 
oder was ist es?
Ist es vielleicht das mit den Uhren oder wie die Informationen erstellt, 
übertragen oder ausgewertet werden.

Wenn du nicht weisst was ich will dann frag halt einfach nach, ich werde 
mich dann bemühen es noch ausführlicher zu beschreiben/aufzuzeigen.


 Kurt

von Robert L. (lrlr)


Lesenswert?

lenk nicht schon wieder ab..

ich hab das Beispiel für dich vereinfacht (Uhrenoszillatoren , GPS, PLL 
..) alles nicht notwendig..

auf was genau willst jetzt raus..?

beantworte doch DU die Frage

> wie lange braucht es im Zug für die Strecke zwischen U1 und U2?

(ps. du willst irgendjemanden von irgendwas überzeugen, also muss DU die 
Antworten und "Beweise" liefern, und DU dir die Arbeit machen, ...)

Du glaubst scheinbar immer, wenn jamand deine Quark nicht versteht, dass 
der dann zu Blöd ist, oder wenn jemand nicht mehr Antwortet recht 
hättest...

von Kurt B. (kurt-b)


Lesenswert?

Robert L. schrieb:
> lenk nicht schon wieder ab..
>
> ich hab das Beispiel für dich vereinfacht (Uhrenoszillatoren , GPS, PLL
> ..) alles nicht notwendig..
>
> auf was genau willst jetzt raus..?
>
> beantworte doch DU die Frage
>
Aber Robert, warum denn so aggressiv!
Ist doch nicht notwendig.

Du hast geschrieben:
--------
ich rate mal:


von S nach E, im Zug, für betrachter/messung im Zug 1 Sekunde..
markierung bei 600000

(von extern betrachtet fährt er also 2 sekunden.)
--------

Du hast geraten dass das Signal im Zug 1 sec braucht, es aber die 
Strecke von 600 000 Km zurückgelegt hat.
Das würde bedeuten dass es mit 600 000 KM/s sich über die Strecke bewegt 
hat, es hat sich aber mit 300 000 Km/s bewegt, denn es hat ja innerhalb 
der 2 sec die Strecke von 600 000 Km zurückgelegt, das ist ja auch an 
der Markierung zu sehen.


>> wie lange braucht es im Zug für die Strecke zwischen U1 und U2?
>
> (ps. du willst irgendjemanden von irgendwas überzeugen, also muss DU die
> Antworten und "Beweise" liefern, und DU dir die Arbeit machen, ...)
>

Ich hab damit keine Probleme, die Arbeit ist ja schon gemacht, und da 
siehst dus ja selber dass deine Raterei nicht stimmen kann, denn das 
Signal kann nicht 1 und 2 sec unterwegs sein, es sei denn du misst mit 
unterschiedlichen Masstäben/Sekunden.
Und das wollen wir doch nicht, oder?

Ich hab Klarheit geschaffen, was missfällt dir daran?


> Du glaubst scheinbar immer, wenn jamand deine Quark nicht versteht, dass
> der dann zu Blöd ist, oder wenn jemand nicht mehr Antwortet recht
> hättest...


Ich hab doch noch garnichts gesagt was du oder andere als richtig oder 
falsch beurteilen könnten.

Aber das will ich gerne machen.
Das gesendete Signal braucht im Zug und ausserhalb des Zuges zwei s bis 
es beim E ankommt.
Wie das gemessen wird, im Zug und ausserhalb, das ist doch auch klar, es 
wird mit Uhren gemessen die dazu in der Lage sind.
Die Strecken sind bekannt, die Laufzeiten und somit auch die 
Geschwindigkeiten auch.
Und da ergibt sich halt dass es im Zug, aus Sicht eines Mitfahrenden im 
Zug, dass das Lichtsignal nicht mit 300 000 Km/s sich im Zug bewegt, 
sondern langsamer.


 Kurt

von Robert L. (lrlr)


Lesenswert?

vergiss es..
ich weiß was du hier falsch betrachtest.. das genügt mir..
ob du es kapierst oder nicht ist mir egal..

: Bearbeitet durch User
von Kurt B. (kurt-b)


Lesenswert?

Robert L. schrieb:
> vergiss es..
> ich weiß was du hier falsch betrachtest.. das genügt mir..
> ob du es kapierst oder nicht ist mir egal..


Leg halt dar was ich falsch betrachte, dann schauen wirs gemeinsam an.


 Kurt

von Robert L. (lrlr)


Lesenswert?

sinnlos.. ciao

von Frank B. (f-baer)


Lesenswert?

Du redest hier von verschiedenen Inertialsystemen. Damit ist deine ganze 
Argumentation hinfällig.
Du bringst ein Gedankenexperiment zur Zeitdilatation, die du ja ohne 
Begründung ablehnst. Du erwartest jetzt aber, dass dir jemand erklärt, 
wie das geht, ohne dabei auf die Zeitdilatation zurückzugreifen. Kannst 
du es?

Du behauptest jetzt, dass aufgrund der "Ortsumstände" die 
Lichtgeschwindigkeit nicht konstant wäre. Du kannst aber die Abstände 
zwischen Zug und Beobachter beliebig klein machen, sogar einen 
röhrenförmigen Zug annehmen, der sich um einen schwebenden ruhenden 
Beobachter herum bewegt und wirst zum selben Ergebnis kommen.
Dann stellt sich doch die Frage, wie lokal deine Ortsumstände wirklich 
sind.
Du kannst das auch auf nahezu masselose Elementarteilchen runterbrechen 
und wirst zum selben Ergebnis kommen. Am Ende des Gedankenexperimentes 
steht immer die Zeitdilatation.

Deine Schlussfolgerung: Im lokalen Kontext eines Inertialsystems sei die 
Lichtgeschwindigkeit konstant. Im Kontext mehrerer zueinander bewegter 
Inertialsysteme sei die Lichtgeschwindigkeit NICHT konstant. Diesen 
Widerspruch kannst du nicht auflösen. Effektiv ergibt sich damit, dass 
die Ausbreitungsgeschwindigkeit des Lichtes selbst dann unbestimmt ist, 
wenn man sie misst, wie du ja erfolgreich dargelegt hast. Um diesen 
Widerspruch aufzulösen sind Zeitdilatation, Längenkontraktion und als 
anschauliches Beispiel Lichtuhr die Stichworte, die du jetzt in der 
Suchmaschine deiner Wahl eingeben darfst.

von Frank M. (ukw) (Moderator) Benutzerseite


Lesenswert?

Frank Bär schrieb:
> Du redest hier von verschiedenen Inertialsystemen.

Kurt lehnt Inertialsysteme ab. Daher kannst Du jedwede weitere 
Diskussion vergessen.

Ich habe schon vor ein paar Tagen meine Konsequenzen gezogen und werde 
auf den irrsinnigen Unsinn von Kurt nicht mehr antworten. Man redet mit 
einer Wand. Auch Robert hat den einzig richtigen Schluss gezogen, 
nämlich auch nicht mehr zu reagieren.

Lass es. Es bringt nichts. Wenn Du nach Kurt googelst, wirst Du sehen, 
dass er diese unsinnigen Diskussionen schon seit Jahren(!) führt und er 
noch NIE irgendetwas angenommen hat.

Kurt stiehlt Zeit. Nämlich unsere.

von Kurt B. (kurt-b)


Lesenswert?

Frank Bär schrieb:
> Du redest hier von verschiedenen Inertialsystemen. Damit ist deine ganze
> Argumentation hinfällig.

> Du bringst ein Gedankenexperiment zur Zeitdilatation, die du ja ohne
> Begründung ablehnst. Du erwartest jetzt aber, dass dir jemand erklärt,
> wie das geht, ohne dabei auf die Zeitdilatation zurückzugreifen. Kannst
> du es?
>
> Du behauptest jetzt, dass aufgrund der "Ortsumstände" die
> Lichtgeschwindigkeit nicht konstant wäre. Du kannst aber die Abstände

...
> Deine Schlussfolgerung: Im lokalen Kontext eines Inertialsystems sei die
> Lichtgeschwindigkeit konstant.

> Im Kontext mehrerer zueinander bewegter
> Inertialsysteme sei die Lichtgeschwindigkeit NICHT konstant. Diesen
> Widerspruch kannst du nicht auflösen.

Hallo Frank Bär,

du kennst doch die ASCII-Zeichnung und deren Erklärungen und sich daraus 
ergebenden Schlussfolgerungen.

Diese sind:

Auf der Erdoberfläche, also dem Gleiskörper, läuft das Signal mit 300 
000 Km/s.
Im Zug, auf der Messtrecke im Zug, läuft es mit einer anderen 
Geschwindigkeit (Angabe auf den Zug bezogen).

Damit die Angaben zu Geschwindigkeit vergleichbar werden (und nicht 
Birnen mit Äpfeln vergleichen) habe ich die Zeiteinheit die zur Aussage 
zu Geschwindigkeiten führt/verwendet wird, festgelegt, diese ist im Zug 
und neben dem Zug identisch lang, nämlich eine Sekunde, die Sekunde die 
auch im Alltag abgewendet wird.

Das Signal ist im Zug und auf dem Gleis 600 000 Km unterwegs, es kommt 
nach zwei Sekunden beim Zugende und beim Gleispunkt an, es ist im Zug 
und ausserhalb des Zuges mit 300 000 Km/s gelaufen (Bezug 
Erdoberfläche).
Aus Sicht des Zuges, der ja mit 150 000 Km/s dem Signal davonläuft halt 
entsprechend langsamer, nämlich mit 150 000 Km/s.

 Kurt

: Bearbeitet durch User
von Kurt B. (kurt-b)


Lesenswert?

Frank M. schrieb:
> Frank Bär schrieb:
>> Du redest hier von verschiedenen Inertialsystemen.
>
> Kurt lehnt Inertialsysteme ab. Daher kannst Du jedwede weitere
> Diskussion vergessen.

Warum sollte ich das, ich weiss das es keine gibt.
Und somit ist klar das alles was auf IS aufgebaut ist ebenfalls nicht 
existiert.


>...

> Kurt stiehlt Zeit. Nämlich unsere.

Tja, das wirst du aber nicht aufrechterhalten können, denn es gibt 
nämlich keine.


 Kurt

Nachfrage: wie schnell bewegt sich das Signal im Zug?
Welche Wellenlänge hat es wenn es sich um ein 100 MHz-Signal handelt?

.
.

: Bearbeitet durch User
von Kurt B. (kurt-b)


Lesenswert?

Robert L. schrieb:
> sinnlos.. ciao

Sieht fast so aus.
Macht aber nichts, vieles ist sinnlos.
(und ist nur auf -Glauben- aufgebaut, gegen dieses Ding ist aber 
wirklich kein Kraut gewachsen)


 Kurt

von Kurt B. (kurt-b)


Lesenswert?

Kurt Bindl schrieb:
> Robert L. schrieb:
>> sinnlos.. ciao
>
> Sieht fast so aus.
> Macht aber nichts, vieles ist sinnlos.
> (und ist nur auf -Glauben- aufgebaut, gegen dieses Ding ist aber
> wirklich kein Kraut gewachsen)
>
>
>  Kurt

Der Zugführer macht wohl Urlaub.
Was ist nun mit dem Polfilter für Licht?
Ist allen klar dass die Erklärungen der -Bücher- dafür physikalisch 
nicht stimmen kann/nicht haltbar ist, denn wieso sollte ausgerechnet 
eine Schwingungsebene, die -elektrische- durch die Gitterstäbe gehen, 
die andere aber nicht von diesen gebremst werden, schliesslich liegen 
sie ja angeblich 90° verschoben vor.
Ausserdem müsste das Durchlassen ja Amplitudenabhängig, also von der 
Helligkeit abhängig sein.
Ist aber nicht, denn die Abhängigkeit besteht darin dass die gestockten 
Antennen/Lichtempfänger entweder angeregt werden oder nicht.
Werden sies dann ists hinterhalb dunkel denn das Signal wird in den 
vielen Resonanzkörpern verbraten und halt einfach zu Wärme.

Frage: ist noch irgendwer der Ansicht dass mit "Longitudinalwellen" 
keine Polarisation möglich ist?
Wenn ja, bitte aufzeigen wieso/warum/weshalb.

 Kurt

von Frank B. (f-baer)


Lesenswert?

Kurt Bindl schrieb:
> Wenn ja, bitte aufzeigen wieso/warum/weshalb.
>
>  Kurt

Warum stellst du an andere Anforderungen, die du selbst nicht willens 
bist, einzuhalten? Deine Axiome sind sich selbst Beweis genug. Hier 
scheint jedenfalls keiner mehr Lust zu haben, mit dir zu diskutieren.

von Frank M. (ukw) (Moderator) Benutzerseite


Lesenswert?

Frank Bär schrieb:
> Warum stellst du an andere Anforderungen, die du selbst nicht willens
> bist, einzuhalten?

Lass es besser, ihm überhaupt noch zu antworten.

Kurt hat bestimmte Methoden (erst Betteln, dann Winseln, dann 
Provozieren, am Schluss dann Auffordern), um diejenigen, die längst 
aufgehört haben mit den Antworten, wieder an die Leine zu kriegen.

Einfach Klappe halten und aushungern lassen. In der Beziehung hat Kurt 
viel mit Trollen gemeinsam.

von Kurt B. (kurt-b)


Lesenswert?

Frank Bär schrieb:
> Kurt Bindl schrieb:
>> Wenn ja, bitte aufzeigen wieso/warum/weshalb.
>>
>>  Kurt
>
> Warum stellst du an andere Anforderungen, die du selbst nicht willens
> bist, einzuhalten? Deine Axiome sind sich selbst Beweis genug.

Darf ich nicht an andere Anforderungen stellen wenn diese den 
"Postulaten" und solche gibt's ja wahrlich genug, den Mund reden und ich 
dann einfach eine Feststellung mache die diese ad Absurdum führen.


> Hier
> scheint jedenfalls keiner mehr Lust zu haben, mit dir zu diskutieren.

Hab ich nichts dagegen, aber wenn keine Antworten auf Einwände kommen 
die aufzeigen dass das was so alles aufgezeigt wird, logischer Unsinn 
ist dann erwarte ich schon Antworten.
Ich rede vom Polfilter für Licht und der Erklärung zu den Gitterstäben.


 Kurt

von Kurt B. (kurt-b)


Lesenswert?

Frank M. schrieb:
> Frank Bär schrieb:
>> Warum stellst du an andere Anforderungen, die du selbst nicht willens
>> bist, einzuhalten?
>
> Lass es besser, ihm überhaupt noch zu antworten.
>
> Kurt hat bestimmte Methoden (erst Betteln, dann Winseln, dann
> Provozieren, am Schluss dann Auffordern), um diejenigen, die längst
> aufgehört haben mit den Antworten, wieder an die Leine zu kriegen.
>
> Einfach Klappe halten und aushungern lassen. In der Beziehung hat Kurt
> viel mit Trollen gemeinsam.

Du kannst es sicher auch viel einfacher sagen dass du keine Antworten 
hast.

 Kurt

von Chris D. (myfairtux) (Moderator) Benutzerseite


Lesenswert?

Richtig.

Kurt, Du hast eben Recht und wir anderen müssen uns das eingestehen.

Da das aber für Männer nicht so einfach ist, werden wir einige Zeit in 
uns gehen müssen.

Daher werden hier auch sehr lange keine Antworten mehr kommen.

von Kurt B. (kurt-b)


Lesenswert?

Chris D. schrieb:
> Richtig.
>
> Kurt, Du hast eben Recht und wir anderen müssen uns das eingestehen.
>
> Da das aber für Männer nicht so einfach ist, werden wir einige Zeit in
> uns gehen müssen.
>
> Daher werden hier auch sehr lange keine Antworten mehr kommen.

Na dann geh mal, den Rückweg wirst du wohl von alleine wieder finden.

Ist man denn wirklich ein Aussätziger und Nestbeschmutzer... wenn man 
den Märchen die einem da verklickert werden widerspricht, ihr 
Märchenhaftes aufzeigt?

Schau dir das Polfilter für Licht an, schau wie es das Signal in Wärme 
umsetzen kann wenn es darauf reagiert, schau welchen 
Bandbreite/Arbeitsbereich so ein Filter hat, schau wie die Moleküle 
ausgerichtet und angeordnet werden damits überhaupt ein Polfilter mimt, 
und dann überlege was passiert wenn polarisiertes/unpolarisiertes Licht 
ankommt.

Dass du dann dein Verständnis für/über Polarisation, was das überhaupt 
ist, ein wenig anpassen musst das wird dann wohl nicht ausbleiben.

(nochmal, wie sollte denn das gehen wenn zwei, senkrecht aufeinander 
stehende Signal vorhanden sind, eine Ebene wird wohl weggedacht damit 
das mit den Gitterstäben verklickert werden kann)

 Kurt

Bitte melde dich an um einen Beitrag zu schreiben. Anmeldung ist kostenlos und dauert nur eine Minute.
Bestehender Account
Schon ein Account bei Google/GoogleMail? Keine Anmeldung erforderlich!
Mit Google-Account einloggen
Noch kein Account? Hier anmelden.